Vous êtes sur la page 1sur 35

Obstetrics,

Gynaecology
and Reproductive
Medicine
Obstetrics, Gynaecology and Reproductive Medicine is a great revision guide for the MRCOG and beyond.
Obstetrics, Gynaecology and Reproductive Medicine is an authoritative and comprehensive resource that provides all obstetricians,
gynaecologists and specialists in reproductive medicine with ready access to up-to-date information on all aspects of obstetrics
and gynaecology. Over a three-year cycle of 36 issues, the emphasis of the journal is on the clear and concise presentation of
information of direct clinical relevance to specialists in the field and candidates studying for MRCOG Part II. Each volume contains
review articles on obstetric and gynaecological topics. The journal is invaluable for specialists in obstetrics and gynaecology,
both in their role as trainers of MRCOG candidates and in keeping up to date across the broad span of the subject area. Over
any three-year period, a subscription will ensure access to up-to-date, understandable information on the full range of obstetrics,
gynaecology and reproductive medicine topics.

Editor-in-Chief
Alec McEwan BA BM BCh MD MRCOG
Consultant in Fetal and Maternal Medicine, Department of Obstetrics and Gynaecology,
Queens Medical Centre, Nottingham, UK

Associate Editors
Sabaratnam Arulkumaran MBBS MD PhD FRCS (Ed) FRCOG
Professor of Obstetrics and Gynaecology,
Department of Obstetrics and Gynaecology,
St. Georges Hospital Medical School, London, UK

Tahir A Mahmood MD FRCOG FRCPI MBA FACOG(Hon)


Consultant Obstetrician and Gynaecologist,
Victoria Hospital, Kirkcaldy, Fife, UK

Philip N Baker FRCOG, FMedSci


Pro-Vice-Chancellor and Head of the College of Medicine,
Biological Sciences and Psychology,
Dean of the School of Medicine, University of Leicester, UK

Fiona Reid MD MRCOG


Consultant Urogynaecologist,
St Marys Hospital, Manchester, UK

Shreelata Datta BSc(Hons) MBBS MRCOG LLM


Consultant Obstetrician and Gynaecologist,
Kings College Hospital, London, UK

Mahmood I Shafi MB BCh MD DA FRCOG


Consultant Gynaecological Surgeon and Oncologist,
Addenbrookes Hospital, Cambridge, UK

Trainee Editor
Catherine Aiken MB/BChir MA PhD MRCP MRCOG
Specialist Registrar (ST5) and Academic Clinical Lecturer in Obstetrics and Gynaecology,
Addenbrookes Hospital, Cambridge, UK
Obstetrics, Gynaecology and Reproductive Medicine has an eminent editorial board, all of whom are recognized experts in their field.
Visit our website at: www.obstetrics-gynaecology-journal.com for previous issues, subscription information and further details.

CASE-BASED LEARNING

Adolescent gynaecology

This adolescent has presented with clinical findings


commonly associated with polycystic ovary syndrome (PCOS).
As many as 5e15% of teens and women are affected by PCOS.
Polycystic ovary syndrome most commonly begins during
adolescence and presents with irregular menstrual cycles and
hyperandrogenism. Difficulties with infertility and metabolic
complications often develop over time. Polycystic ovary syndrome is diagnosed in adults using the Rotterdam consensus
criteria. Two of the following criteria are required for a diagnosis
of PCOS in adults:
 Oligo- or anovulation
 Clinical and/or biochemical signs of hyperandrogenism
 Polycystic ovaries
These criteria require special interpretation in adolescents.
Polycystic ovaries contain 12 or more follicles measuring 2e9
mm in diameter in each ovary, or an ovarian volume greater than
10 mL on abdominal ultrasound in adult women. Similar criteria
are sometimes used, but not well supported in adolescents.
Ovaries generally remain stable in size, less than 2 cm3, in girls
from 2 to 9 years of age, and progressively increase in size during
adolescence. Follicle counts in adolescents are higher than in
adults, and can lead to a misdiagnosis if the Rotterdam criteria
are used. Best ovarian visualization is obtained with vaginal ultrasound in adults, but transabdominal ultrasound is used to
visualize the ovaries in girls who are not sexually active. Girls
experiencing regular menstruation should be scanned in the early
follicular phase (cycle days 3e5), while those not yet experiencing menstruation can be evaluated at any time.
The 1990 Consensus Conference of the National Institute on
Child Health and Human Development based the diagnosis of
PCOS in adolescents on the presence of anovulation and hyperandrogenism, not explained by other endocrine disorders. Adolescents with polycystic ovary syndrome (PCOS) often present at
menarche with irregular and long menstrual cycles. Adolescents
with two years of irregular cycles following the onset of
menarche should be evaluated for PCOS. Irregular cycles can
occur from 45 days to a year apart and are generally heavy and
associated with cramping pain. Adolescents with regular menstrual cycles and hyperandrogenism and/or polycystic ovaries
should also be evaluated for PCOS. Many normal adolescents
have irregular cycles regulated with oral contraceptives. Adolescents with undiagnosed PCOS treated with oral contraceptives
to regulate their cycle can remain undiagnosed until they seek
evaluation for infertility.
Dysfunctional uterine bleeding (DUB) is defined as irregular
uterine bleeding with no pelvic or general medical disease, or
pregnancy. Affected patients have a loss of the normal cyclic
pattern of ovulatory luteinizing hormone (LH) surge in response
to rising oestradiol levels during ovulation. Endometrium in
these girls continues to proliferate under the influence of unopposed oestrogens, and does not undergo secretory change.
Endometrial shedding in these girls is usually prolonged and
heavy. DUB is observed in anovulatory and oligo-ovulatory
women with PCOS. About 1e2% of women with anovulatory
bleeding, who are not properly treated, would develop endometrial cancer.
Decreased insulin-mediated glucose utilisation, aka insulin
resistance, is found in as many as half of affected patients. Girls
with PCOS have a three to seven times increased risk of

Mohamed Otify
Tahir Mahmood

Abstract
Paediatric and adolescent gynaecology involves the care of young
women and their special needs as their bodies change and grow
into adulthood. Diagnosis and management of common problems in
gynaecology in this age group requires knowledge regarding congenital reproductive anomalies, disorders of gender development, reproductive endocrinopathies, gender identity, and gynaecologic
malignancies. Gynaecologic disease states in paediatric patients
frequently present with vulval and vaginal manifestations, while
those in the adolescent age range present as abdominal-pelvic pain
and abnormal menstrual bleeding. A case-based learning of common
childhood gynaecologic disorders and their treatment is presented.

Keywords dysfunctional uterine bleeding; immature malignant


teratoma; imperforate hymen; ovarian mass; polycystic ovary
syndrome; precocious puberty; primary amenorrhoea

Introduction
Paediatric and adolescent gynaecology requires knowledge of
normal female embryological development, endocrine function
around adolescent years, fertility, the impact of female genital
system abnormalities on gender identity and mental health, and
the medical and surgical management of these systems. Best care
is administered with a multidisciplinary team (including
adolescent gynaecologists, paediatricians, paediatric surgeons,
endocrinologist, clinical geneticists and psychologists) to treat
the underlying disorder, preserve fertility and future sexual
function, and maintain a healthy mental state. This review presents different scenarios in children presenting with a diverse
range of gynaecological diseases requiring knowledge of these
systems.

Case 1: polycystic ovary syndrome and dysfunctional


uterine bleeding
A 14-year-old girl presents with irregular menses since her
menarche at the age of 11. Her menstrual flow usually lasts from
4 to 6 days, is heavy, and is associated with painful cramping.
The intercycle interval varies between 40 and 60 days. Her
mothers onset of menarche occurred at 12 years of age. The girl
also complains of increased acne and facial hair over the last year
accompanied by weight gain disproportionate to her growth in
height.

Mohamed Otify MSc MRCOG is a Specialty Trainee in Obstetrics and


Gynaecology at Royal Inrmary of Edinburgh, Edinburgh, UK.
Conicts of interest: none declared.
Tahir Mahmood MD FRCPI FACOG MBA FRCPE FRCOG is a Consultant
Obstetrician and Gynaecologist at Victoria Hospital, Kirkcaldy, Fife,
UK. Conicts of interest: none declared.

OBSTETRICS, GYNAECOLOGY AND REPRODUCTIVE MEDICINE 26:7

216

2016 Elsevier Ltd. All rights reserved.

CASE-BASED LEARNING

developing type 2 diabetes. This risk is greatest in girls with


anovulatory cycles, those with polycystic ovaries, obese girls,
and girls with a family history of type 2 diabetes. The presence of
altered lipid metabolism suggests these girls may be at increased
risk for cardiac disease as well.

hyperthyroidism in women with PCOS is not felt to be greater


than that of the general population and the use of routine thyroid
function determination is not recommended. Prolactin secretion
can increase the production of androgens by the adrenal glands
and prolactin levels are generally determined.

Clinical ndings
Relevant history includes age at menarche, breast bud development, and appearance of pubic hair, duration of periods, amount
of menstrual flow, presence of pain or cramping, time between
periods, diet, and exercise patterns. Girls with PCOS most
frequently have regular periods to start and then develop irregular periods. Growth charts of adolescents with PCOS can
demonstrate a marked increase in weight following initiation of
menses. A family history for PCOS and diabetes is also obtained.
Adolescents with PCOS and hyperandrogenism can demonstrate Hirsuitism and virilisation. Hirsuitism is characterized by
excess facial and body hair. The frequency of Hirsuitism is
directly related to age, with adolescents being least commonly
affected. Signs of virilisation include deep voice, frontal and
temporal balding, increased muscle bulk, decreased breast size,
and clitoral hypertrophy. The clinical finding of acne is generally
normal during adolescence and can be confusing in virilised
adolescents. Acanthosis nigricans, characterized by darkening of
the skin on the back of the neck and other areas, is associated
with moderate to severe insulin resistance.
At first clinic appointment, patient height, weight, BMI, waist
circumference, and blood pressure are evaluated. Affected adolescents generally have a central obesity pattern.

Treatment
Girls with PCOS require family and personal counselling to better
understand the long-term impact of this diagnosis. Depression
and anxiety can develop as the patient experiences the clinical
impact of their disease. The treatment of PCOS is symptomatic.
General recommendations include, diet, exercise, and weight
reduction, which can improve serum androgen, insulin, LH and
lipid abnormalities. Further treatment may be needed if the
adolescent woman with treated PCOS desires pregnancy.
Medroxyprogesterone or Norethisterone administered for 10
e14 days each month can be used to avoid abnormal endometrial proliferation and facilitate normal ovarian androgen production in adolescents. The low-dose oral contraceptives pills
can also be used in girls with regular menses. Such treatment will
provide contraception, prevent endometrial hyperplasia,
decrease testosterone levels, and can improve hirsuitism and
acne. Adolescents with irregular menstrual cycles may also be
started on oral contraceptives without evaluation for PCOS. Girls
already taking oral contraceptives can be withdrawn for 3
months if biochemical testing is needed to diagnose hyperandrogenism or PCOS. Other contraceptive measures e.g. barrier
contraception should be planned for during this testing period,
when appropriate.
The use of metformin has been beneficial in increasing insulin
sensitivity and decreasing serum LH and free testosterone levels.
The use of metformin has been associated with the return of
normal menstrual cycles in 68e95% of girls with prior normal
cycles.
Antiandrogens such as spironolactone, flutamide, or cyproterone can be combined with oral contraceptive pills to decrease
testosterone levels and treat hirsutism. GnRH analogues are
generally reserved for girls that do not respond to these treatments or do not tolerate oral contraceptive pills. Long-term use of
GnRH analogy can lead to bone demineralization, hot flushes,
and atrophic vaginitis due to chronic low oestrogen levels, and
may require monitoring as long as they are on this treatment.

Laboratory testing
Patients are diagnosed with PCOS after excluding the diagnosis of
congenital adrenal hyperplasia, androgen-secreting tumours,
high-dose exogenous androgens, and Cushings syndrome. A
basal morning 17-hydroxyprogesterone level is determined in
order to exclude 21-hydroxylase deficient non-classic adrenal
hyperplasia in hyperandrogenic patients.
Serum follicle stimulating hormone (FSH) and 17b-oestradiol
levels are determined to exclude hypogonadotropic hypogonadism in young women presenting with oligo- or anovulation.
The free androgen index (total testosterone level divided by
sex hormone-binding globulin level) and serum dehydroepiandrosteronesulphate (DHEAS) levels are used to identify
biochemical hyperandrogenism. A dexamethasone suppression
test is performed to exclude Cushings syndrome.
At subsequent review appointments, consideration should be
given to assess fasting serum glucose level, total, and low-density
lipoprotein and high-density lipoprotein cholesterol levels to
determine insulin resistance. Oral glucose tolerance testing is
performed as indicated.
Luteinizing Hormone (LH) levels are significantly elevated in
about 60% of girls with PCOS, due to the increased amplitude
and frequency of LH pulses. The LH/FSH ratio is elevated in
about 95% of affected patients. The clinical relevance of LH in
PCOS and the potential effects of LH suppression with gonadotropin releasing hormone (GnRH) analogues are not known, so
LH levels are not routinely determined.
Thyroid hormones are known to alter androgen metabolism
and contribute to hyperandrogenism. However, the incidence of

OBSTETRICS, GYNAECOLOGY AND REPRODUCTIVE MEDICINE 26:7

Case 2: ovarian mass


A 15-year-old girl presents with acute peri-umbilical pain,
palpable abdominal mass, nausea, vomiting, and radiation of
pain to the groin. Pelvic ultrasound demonstrates a 17 cm complex pelvic mass. Plain computed tomography (CT) imaging
demonstrates a large, heterogeneous mass with a prominent
solid component, calcifications, and small foci of fat.
As many as 10% of women undergo evaluation for an ovarian
mass during their life. Almost all ovarian masses in premenopausal females are benign. Pre-pubertal adolescents develop
functional cysts due to the failure of involution of follicles. These
cysts are caused by gonadotropin stimulation of the ovary by the
immature hypothalamicepituitary axis. Benign ovarian cysts/or
immature follicular cysts, less than 1 cm in diameter, have been
reported in 2e5% of pre-pubertal females undergoing

217

2016 Elsevier Ltd. All rights reserved.

CASE-BASED LEARNING

A pregnancy test and complete blood count are obtained.


Lactate dehydrogenase (LH), a-FP, and b-HCG levels are determined in all women less than 40 years of age with complex
ovarian masses in order to stage germ cell tumours and to aid
follow-up detection of any recurrences after treatment.
Elevated serum CA-125 levels are indicative of epithelial
ovarian cancer in adult woman, but are unreliable in prepubertal girls due to their poor specificity. A serum CA-125 level
is not routinely obtained when pelvic ultrasound demonstrates a
simple ovarian cyst. Several algorithms have been developed to
better identify malignant ovarian lesions, but are not useful in
adolescents due to the occurrence of non-specific elevations in
CA-125 in these patients.

ultrasound. Post-pubertal adolescents develop cysts after a failure of ovulation or with the persistence of ovarian follicles.
About one in 1000 symptomatic ovarian cysts in premenopausal females are malignant, increasing to about three in 1000
at age 50. The incidence of adnexal masses in girls less than 18
years of age has been estimated as 2.6 per 100,000. Ten percent
of ovarian masses have been reported to be non-ovarian in
origin. Benign mature teratomas (dermoid cysts) are the most
common benign tumour in girls less than 20 years of age. About
10% of ovarian masses in this age group are malignant. Malignant immature teratomas comprise less than 1% of ovarian teratomas and 10e20% of malignant tumours in this age range.
Malignant immature teratomas comprise about 10% of germ
cell tumours. Germ cell tumours can be associated with elevations in serum a-fetoprotein (a-FP) and b-human chorionic
gonadotropin (b-hCG) levels. Elevated serum a-FP levels have
been reported in 33e65% of patients with malignant immature
teratoma. About 26% of affected patients also have associated
ipsilateral mature cystic teratoma and 10% contralateral immature teratoma. Malignant immature teratomas are diagnosed
most frequently between the ages of 10 and 20 years, with a
median age of 17 years.

Treatment
Patients are treated according to tumour stage. Clinical staging
for malignant immature teratoma is:
 Stage 1: tumour confined to the ovary
 Stage 2: local tumour extension to the fallopian tubes,
uterus, or other pelvic organs
 Stage 3: tumour spread confined to the peritoneum or
abdominal lymph nodes
 Stage 4: tumour spread outside the peritoneal cavity
Simple ovarian cysts less than 50 mm in diameter are usually
functional and resolve over two to three menstrual cycles without
intervention. Simple ovarian cysts 50e70 mm in diameter are
followed with yearly ultrasound to identify any change. Larger
simple cysts are usually evaluated by using MRI or laparoscopic
surgery. The use of oral contraceptives has not been helpful in
treating functional ovarian cysts. Oral contraceptives have been
useful in post-pubertal females for the prevention of ovarian cysts.
Ovarian cysts that persist or increase in size during follow-up
are usually not functional cysts and are managed surgically.
Mini-laparotomy may be used to resect very large cysts or solid
tumours. Adolescents and their parents should be counselled
regarding the possibility of ovarian loss with surgery.
Malignant immature teratomas are usually unilateral and are
diagnosed with laparoscopic oophorectomy of the affected
gonad. Patients with stage 1 malignant immature teratomas are
treated with oophorectomy and observation. Ovary sparing surgery is performed when possible in order to better preserve
fertility. Visual inspection of the abdominal contents is performed during surgery to identify malignant spread. Pelvic
washings, inspection of the contralateral ovary, and intraoperative frozen section are also performed. Patients with large
tumours are converted to open surgery to better prevent tumour
spillage and to ease removal. The benefit of lymphadenectomy in
women with lymphadenopathy is not clear.
Patients with stage 2, 3, and 4 malignant immature teratomas
are treated with ovarian surgery plus chemotherapy using bleomycin, etoposide and cisplatin (BEP). Limiting chemotherapy
related toxicities include pulmonary toxicity with bleomycin,
decreases in glomerular filtration rate and high tone hearing loss
with cumulative doses of platinum exceeding 300e400 mg/m2,
and the late development of myeloid malignancies with
etoposide. The rarity of these tumours has slowed the evaluation
of chemotherapy in randomized controlled clinical trials.
Fertility after conservative surgery and chemotherapy is found
in 70e83.3% of adolescents treated with conservative surgery

Clinical ndings
Adolescents with ovarian masses can present with abdominal
pain, nausea, vomiting, a palpable abdominal mass, increasing
abdominal girth, or precocious puberty. Larger ovarian tumours
are predisposed to torsion, which typically present with a sudden
onset of severe abdominal pain, fever, nausea, vomiting, and
leucocytosis.
A thorough history and physical exam are performed to
evaluate genitourinary and gastrointestinal symptoms. Physical
exam should take special care to identify lymphadenopathy,
abdominal tenderness, and the presence of ascites. Menstrual
cycle, sexual, and contraceptive history are obtained. Any relationship of symptoms to menstrual cycle is determined. Family
history for ovarian or breast cancer is obtained. Symptoms suggestive of ovarian malignancy are identified, including history of
persistent abdominal distension, change in appetite, pelvic or
abdominal pain, increased urinary urgency and/or frequency,
and weight loss.
Laboratory testing
Pelvic ultrasound is performed to identify and characterize
ovarian tumours and cysts. Sensitivity of pelvic ultrasound has
been reported as 95%, specificity as 91%, positive likelihood
ratio as 10.4 and negative likelihood ratio as 0.06.
Chest and abdominal imaging with CT or magnetic resonance
(MRI) are useful in further staging adolescents with complex or
solid ovarian tumours. Malignant immature teratoma usually
present as a large, heterogeneous mass with a prominent solid
component on imaging studies. Calcifications and small foci of
fat are often present. The presence of these three findings in an
adolescent is suggestive of malignant immature teratoma. Haemorrhage is occasionally seen. Identification of a solid portion
with numerous cystic areas is helpful in distinguishing malignant
immature teratoma from benign mature cystic teratoma. Benign
mature teratomas are usually smaller and contain more cystic
areas than malignant immature teratoma.

OBSTETRICS, GYNAECOLOGY AND REPRODUCTIVE MEDICINE 26:7

218

2016 Elsevier Ltd. All rights reserved.

CASE-BASED LEARNING

and chemotherapy, and no fetal abnormalities have been reported. Radiation therapy is avoided in order to preserve fertility,
unless there is residual tumour after second-line chemotherapy
and surgery.
The long-term recurrence rate after surgical excision of
mature cystic teratomas is around 4%. A patient with young age
(<30 years old) or large cyst (8 cm in diameter) or bilateral
cysts is at high risk of recurrence, which is even higher when a
patient has more than one of these factors.
In cases of immature teratomas recurrence was reported in
18% of the patients with grade 1 disease, in 37% of the patients
with grade 2 disease, and in 70% of the patients with grade 3
diseases.

hypothyroidism, drugs such as phenothiazines, and with pituitary


prolactinomas. Gonadal failure can be due to premature ovarian
failure, similar to that found with menopause or with ovarian
dysgenesis (Turners syndrome).
Clinical ndings
A family history of delayed or absent puberty and family height is
obtained. A history of difficulties after birth can indicate
congenital adrenal hyperplasia. A history of drugs and their side
effects are evaluated.
Signs of pubertal development and presence of normal internal structures are documented. Signs of hyperandrogenism are
compatible with PCOS. Signs of virilisation are compatible with
more severe androgen excess found with androgen-secreting
ovarian or adrenal tumours or 5-alphareductase deficiency.
Breast development is assessed by Tanner staging. The presence
or absence of breast development is used as a marker for oestrogen action and ovarian function. Galactorrhoea is found in
patients with prolactin secreting pituitary tumours or drugs.
Growth, including height, weight, and arm span, are evaluated.
Patients with Turner syndrome have a characteristic low hairline, webbed neck, shield chest, and widely spaced nipples. The
blood pressure in both arms is examined to screen for coarctation
of the aorta in these patients. A genital exam characterizing
clitoral size, pubic hair development, intactness of the hymen,
vaginal length, and presence of a cervix, uterus, and ovaries is
performed. A small cotton swab can be used to explore the vagina for determination of its presence and depth.

Case 3: primary amenorrhoea


A 19-year-old girl presents with absence of spontaneous
menstruation and of any breast development. In the past, her
paediatrician has documented lack of a pubertal growth spurt on
yearly physical examinations. The girls mother and two sisters
experienced first menstruation at ages 12e14 years.
Primary amenorrhoea is defined as the failure of menstruation
by age 16 years in the presence of normal secondary sexual characteristics or age 14 years in the absence of other evidence of puberty. The prevalence of amenorrhoea in age appropriate women is
about 1%. The most common causes include chromosomal abnormalities such as gonadal dysgenesis (50%), hypogonadotropic
hypogonadism (20%), Mullerian agenesis (15%), transverse
vaginal septum or imperforate hymen (5%), and pituitary disease
(5%).
Women with secondary sexual characteristics can be pregnant, or have a constitutional delay, genitourinary malformation
ster-Hauser syn(Mullerian agenesis, Mayer-Rokitansky-Ku
drome), PCOS or testicular feminization. Women with a constitutional delay have normal development, but a later onset than
their peers. The age of menarche in their mother or sisters may
also be delayed. Testicular feminization consists of the failure of
normal masculinization of the external genitalia in genetically
male individuals (affecting one in 100,000 live births). Affected
individuals have a male chromosome pattern and an inactivating
mutation in the dihydrotestosterone receptor. Male gonads are
found in the inguinal canal or abdomen along with normal female external genitalia including normal labia, clitoris, and
vaginal introitus.
ster-Hauser syndrome consists of
Mayer-Rokitansky-Ku
congenital absence of the vagina with variable uterine development (affecting about one in 5000 females). Seven to 10% of
affected women have a normal but obstructed or rudimentary
uterus with functional endometrium. The presence of an imperforate hymen or vaginal or uterine agenesis can also result in the
absence of menarche.
Women without secondary sexual characteristics can have
hypothalamic failure, Kallmans Syndrome, hyperprolactinaemia,
or gonadal failure. Hypothalamic failure can occur due to chronic
illness, excessive exercise, stress, extreme malnutrition, or
anorexia nervosa. Kallmans Syndrome can occur due to failure of
secretion of gonadotropin releasing hormone (GnRH) (affecting
one in 125,000 females), the presence of hypothalamic or pituitary
tumours, or hydrocephalus. Hyperprolactinaemia can occur with

OBSTETRICS, GYNAECOLOGY AND REPRODUCTIVE MEDICINE 26:7

Laboratory testing
Standard testing includes a pregnancy test, and determination of
serum FSH, LH, prolactin, total testosterone, sex hormonebinding globulin and thyroid stimulating hormone levels. A
pelvic ultrasound can be useful in defining gonadal and uterine
anatomy.
Elevated serum FSH levels are associated with ovarian dysgenesis, leading to examination of the patients karyotype. A 46 XY
karyotype is associated with a high risk for gonadoblastoma or
dysgerminoma arising from retained testes. A normal serum FSH
level with no visualization of the uterus on ultrasound is compatible
with Mullerian agenesis or androgen insensitivity syndrome. Serum
testosterone levels are in the normal range in women with Mullerian agenesis and in the normal range for genotypic men (phenotypic women) with androgen insensitivity. A normal serum FSH
level with breast development and ultrasound visualization of
blood in the uterus (hematometra) or vagina (hematocolpos) is
compatible with an obstructed outflow tract. A low or normal
serum FSH level with a normal uterus visualized on ultrasound is
compatible with constitutional delay of puberty or congenital GnRH
deficiency. Magnetic resonance imaging (MRI) is the mainstay in
the imaging evaluation of Mullerian agenesis.
Treatment
Treatment is dependent on the nature of the underlying disorder.
Medical treatment is indicated for thyroid disease. Drugs that
cause prolactin secretion are identified, and modified as
indicated.
Patients with Turners syndrome and testicular feminization
have particular health risks and should be referred for specialist

219

2016 Elsevier Ltd. All rights reserved.

CASE-BASED LEARNING

evaluation and care. Surgery may be indicated to correct physical


abnormalities or for the treatment of pituitary tumours. Surgical
removal of the gonads is indicated in patients with a 46 XY
karyotype because of the high risk of tumour formation in
cryptorchid testes.
Patients with gonadal failure will need hormone replacement
therapy until the age of natural menopause. Calcium and vitamin
D supplementation may be indicated to prevent osteoporosis.
Patients with male karyotype will need counselling regarding
the development of normal menstruation, possibility of pregnancy, and the effect of their disease on their gender identity.

syndrome, and hypophosphatemic rickets. Patients are recognized clinically by the triad of peripheral precocious puberty,
irregular cafe-au-lait spots, and fibrous bone dysplasia. Girls with
MAS have recurrent follicular cysts and cyclic menses.
Amenorrhoea can be caused by obstructions in menstrual
outflow. The most common cause of outflow obstruction is an
imperforate hymen. The hymenal membrane separates the
vaginal lumen from the urogenital sinus and usually ruptures
before birth, leaving a thin fold of membrane. New born children
with imperforate hymen have stimulation of vaginal secretions
from maternal oestradiol and a bulging introitus on physical examination due to mucocolpos. The mucous is resorbed as oestradiol levels drop and this finding is not subsequently observed
again until menstruation, when it can present as hematocolpos.

Case 4: precocious puberty


A 7-year-old girl is brought to her paediatrician with complaints
of abdominal pain. Her height and weight are both at the 95th
percentile. Her review of systems is normal. Physical examination reveals Tanner stage 2 pubic hair that the mother says
developed over the past year. She has no breast buds. Vulvar
inspection reveals a tense, shiny, bulging membrane. The childs
mother had her first menses at age 11 years.
The normal age of onset of puberty in girls is about 10.5 years.
Puberty is manifested as breast development (thelarche), followed by pubic hair development (pubarche), and later
menarche. These changes occur over a 2e3 years period. Precocious puberty is defined as sexual development before 8 years
of age, 2.5 e 3 standard deviations below the normal age of onset
of puberty. A more controversial report by the Lawson Wilkins
Paediatric Endocrine Society suggests this age should be lowered
to 6 years in black girls and 7 years in white girls. Normal variations in the onset of puberty include isolated breast development in girls (premature thelarche) or isolated androgenmediated sexual characteristics (such as pubic and/or axillary
hair, acne, and apocrine odour) resulting from early activation of
the hypothalamic pituitary adrenal axis.
Precocious puberty occurs in about one in 5000 children and
is about 10 times more common in girls than in boys. Precocious
puberty can be central or peripheral in nature. Central precocious
puberty (CPP) can be associated with CNS disorders resulting in
premature activation of the hypothalamic pituitary gonadal axis.
Eighty to 90% of girls with central precocious puberty are idiopathic in nature. Peripheral precocious puberty (PPP) is caused
by the secretion of sex hormones associated with gonadal and
adrenal disorders, abdominal or pelvic tumours, or inadvertent
exposure to topical preparations of oestrogen or androgen. Girls
with severe, long-standing primary hypothyroidism will also
occasionally present with precocious puberty.
Genetic causes of precocious puberty include gain of function
mutations in the Kisspeptin 1 gene and its G protein-coupled receptor, KISS1R. Loss of function mutations in KISS1R are associated with hypogonadotropic hypogonadism. Loss of function
mutations in makorin ring finger protein 3 (MKRN3) also result in
precocious puberty. Familial cases of central precocious puberty
have been attributed to paternally transmit inactivating mutations of the MKRN3 gene. Patients with McCuneeAlbright Syndrome (MAS) have a somatic mutation of the alpha subunit of
the G3 protein that activates adenylyl cyclase. This mutation
leads to continued endocrine activation including precocious
puberty, thyrotoxicosis, gigantism or acromegaly, Cushing

OBSTETRICS, GYNAECOLOGY AND REPRODUCTIVE MEDICINE 26:7

Clinical ndings
Children with precocious puberty have accelerated linear growth
for age, advanced bone age, and pubertal levels of serum LH and
FSH. Untreated precocious puberty can result in accelerated
skeletal development and shorter adult height. Physical examination includes evaluation of height, weight, and height velocity
(cm/year), assessment of visual fields, and examination for cafeau-lait spots. Vulvar or vaginal inspection reveals a tense, shiny,
bulging membrane. Secondary sexual maturation is evaluated
using the Tanner stage.
Some children develop central precocious puberty secondary to
CNS pathology, a condition called neurogenic CPP. Lesions causing
neurogenic CPP include hamartomas of the tuber cinereum, astrocytomas, ependymomas, pinealomas, optic and hypothalamic gliomas, hydrocephalus, cysts, trauma, CNS, inflammatory disease,
and congenital midline defects, such as optic nerve hypoplasia.
Children with neurofibromatosis can develop neurogenic CPP with
the occurrence of optic gliomas. Children with CPP should undergo
brain contrast-enhanced MR imaging to exclude such lesions.
Children receiving CNS irradiation can develop precocious puberty
secondary to loss of growth hormone secretion.
The most common cause of PPP in girls is a large functional
ovarian follicular cyst. Affected patients exhibit early breast
development followed by vaginal bleeding due to oestrogen
withdrawal when the cyst regresses. Functional cysts can appear
and regress spontaneously, leading to a conservative management plan. Large cysts have a higher risk of ovarian torsion.
The adolescent girl with an imperforate hymen will present
after menarche with cyclic abdominal or pelvic pain and hematocolpos, a perirectal mass from sequestration of blood in the
vagina, and primary amenorrhoea. Physical exam reveals a
bulging hymenal membrane with a bluish discolouration due to
retained blood. Marked distension of the vagina can lead to back
pain, pain with defecation, and problems with urination.
Transverse vaginal septae can present in a similar fashion.
Laboratory testing
Routine testing includes bone age assessment, LHRH stimulation
testing, and determination of serum FSHF, LH, testosterone, and
oestrogen levels. Pelvic ultrasound is used to assess ovarian and
uterine anatomy. Patients with normal variations in the onset of
puberty can have mildly elevated levels of serum DHEAS for age.
Patients with normal variations in the onset of puberty are closely
followed to identify their complete progression into puberty.

220

2016 Elsevier Ltd. All rights reserved.

CASE-BASED LEARNING

Serum LH levels in the pre-pubertal range are consistent with


PPP or the benign pubertal variant. Higher serum LH levels are
found in children CPP. Girls under the age of two years can have
higher serum LH levels associated with the mini-puberty of infancy. Very high levels of oestradiol and suppressed serum LH
and FSH levels are found in children with PPP, usually from an
ovarian tumour or cyst. Adrenal disorders are identified using
serum 17-hydroxyprogesterone, testosterone, and DHEAS determinations, evaluation of urinary steroid secretion, ACTH
stimulation testing, and CT or MR of the adrenal glands. Children
with ongoing pubertal progression and a pre-pubertal serum LH
level can undergo a GnRH stimulation test to differentiate CPP
from the benign pubertal variant.
Evaluation of skeletal maturation by radiographic assessment
of bone age is useful in diagnosing precocious puberty and
assessing its impact on final patient height. A bone age two
standard deviations greater than chronological age is suggestive
of central or peripheral precocious puberty, rather than a benign
pubertal variant.
A contrast-enhanced brain MR is performed in girls with
central precocious puberty and an onset of secondary sexual
characteristics before six years of age, where the incidence of
abnormal findings may be as high as 20%.Ten older children
have a lower risk of CNS abnormalities and less frequent findings
on MRI. Pelvic ultrasonography shows larger uterine and ovarian
volumes in girls with CPP, compared to girls that are pre-pubertal
or have premature thelarche. A pelvic ultrasound is performed in
girls with PPP to identify ovarian cysts or tumours. An adrenal
ultrasound is performed in girls with rapid virilisation.
Other testing may include thyroid function testing, a CT or MR
of the brain and pituitary, and evaluation for hyperostosis of base
of skull in McCuneeAlbright syndrome. A skeletal survey will
reveal polyostotic fibrous dysplasia in patients with McCune
eAlbright syndrome.

Surgery for imperforate hymen is based on the presence of


symptoms and can be performed in the post-pubertal, or premenarchal time periods, when oestrogen stimulation is highest.
If an asymptomatic patient is diagnosed with an imperforate
hymen without a mucocele during childhood, this patient can be
treated after the onset of puberty and prior to the development of
a hematocolpos or hematometra. The imperforate hymen is
incised in a cruciate fashion, close to the hymenal ring. General
anaesthesia is administered to allow careful incision and complete evacuation of retained blood and mucous. Transverse
vaginal septae are treated in a similar fashion.

Conclusion
The initiation and normal progression of puberty in adolescents
can be affected by numerous diseases and present in varied
ways. The cases presented, PCOS, primary amenorrhoea, and
precocious puberty, well characterize the diversity in findings in
the endocrine systems, clinical history, and physical examination. Characterization of embryologic development, pubertal
development, and associated endocrine abnormalities is performed for each disease state. The lack of age normal standards
for ovarian and uterine development can add to the complexity of
this evaluation. Identification of abnormal findings, such as
virilisation, can be important in making a rapid correct initial
diagnosis. Multidisciplinary input is vital to counselling process
with a team consisting of paediatric endocrinologist, gynaecologist with interest in pre-and adolescent gynaecology, paediatric
surgeon and psychiatrist.
A
FURTHER READING
Azziz R. Controversy in clinical endocrinology: diagnosis of polycystic
ovarian syndrome: the Rotterdam criteria are premature. J Clin
Endocrinol Metab 2006; 91: 781e5.
ger J. Precocious puberty. N Engl J Med 2008; 358:
Carel JC, Le
2366e77.
Devaja OM, Papadopoulos AJ. Current management of immature
teratoma of the ovary. Arch Oncol 2000; 8: 127e30. 6.
Kent SC, Legro RS. Polycystic ovary syndrome in adolescents. Adolesc Med 2002; 13: 73e88.
Ledger WL, Atkin SL, Sathyapalan T. Long-term consequences of polycystic ovary syndrome. Green-top Guideline No. 33 2014; 1e15.
Zolton JR, Maseelall PB. Evaluation of ovarian cysts in adolescents.
Open J Obstet Gynecol 2013; 3: 12e6.

Treatment
Untreated women achieve a mature height of 151 cme155 cm.
Treatment with GnRH agonists is effective in improving adult
height of children diagnosed before 6 years of age. The benefit of
treating girls diagnosed at ages 6e8 years is not clear. Girls in this
age range with advanced bone age or a height below the 25th
percentile with predicted heights less than 150 cm or who are well
below their target height are considered for treatment. No benefit
has been found in treating children presenting at ages greater than
8 years. Best results are achieved in girls without advanced bone
maturation. Treatment should be stopped once an acceptable age
of puberty is reached. Normal gonadotropin secretion recommences about 3e4 months after cessation of treatment, with
normal pubertal development and fertility. Children with precocious puberty due to radiation and growth hormone deficiency
require additional growth hormone supplementation.
The profound physical and hormonal changes associated with
precocious puberty can lead to difficulties in interpersonal relationships, disruptive behaviour, and an early onset of sexual
behaviour. Counselling for the patient and family are indicated.
Treatment outcomes are monitored using age and gender
appropriate growth charts, growth velocity, bone age, pubertal
progression, Tanner staging, and determination of serum gonadotropins and sex steroid levels.

OBSTETRICS, GYNAECOLOGY AND REPRODUCTIVE MEDICINE 26:7

Practice points
C

C
C

221

Polycystic ovarian syndrome is often undiagnosed as many adolescents do not have regular menstrual cycles. Symptomatic
treatment of these young women with oral contraceptive pills can
contribute to missing this diagnosis.
The four most common causes of amenorrhoea are polycystic
ovarian syndrome, hypothalamic amenorrhoea, ovarian failure,
and hyperprolactinaemia.
90% of central precocious puberty in girls is constitutional.
Many adolescents with gynaecologic pathology present with
complex problems that are best managed with a team approach.

2016 Elsevier Ltd. All rights reserved.

REVIEW

Endometrial cancer

features. They are associated with obesity, nulliparity, insulin


resistance, and a hyper-oestrogenic environment e.g. the use of
unopposed oestrogens or ovarian granulosa cell tumour. These
tumours often exhibit mutations in the PTEN tumour suppressor
gene; k-ras oncogene and mismatch repair genes and frequently
stain positively for oestrogen and progesterone receptors.
Type 2 tumours i.e. serous, clear cell, squamous and undifferentiated carcinomas, carcinosarcoma (previously called Malignant Mixed Mullerian Tumour) and endometrial stromal
sarcomas (ESS), are less common, more aggressive and have a
poorer prognosis. They are not associated with the risk factors
for type 1 cancers. Often these tumours occur in older women. At
a molecular level, mutations of the p53 tumour suppressor gene
are common. Commonly, trans-peritoneal dissemination is seen
with a pattern of spread that is reminiscent of ovarian cancers.
Endometrial cancer is usually primary, however in rare cases
may be metastatic from other tumours (e.g. breast, ovary, lung,
stomach, colorectal, and melanoma). Endometrial carcinomas
spread by direct extension to the cervix, vagina and myometrium.
Vaginal metastases (drop-lesions) can also occur as a result of
haematogenous spread. Deeper myometrial invasion eventually
leads to breach of the uterine serosa and parametrial involvement.
The risk of lymph node involvement in endometrial cancer is
directly related to the depth of myometrial invasion as well as the
grade. Lymphatic spread occurs to the external iliac, internal iliac
and obturator lymph nodes in the pelvis, and to the para-aortic
nodal chain. The latter is less common if the pelvic nodes are
not involved, although direct spread via lymphatic channels
draining the upper uterus can occur. Trans-tubal spread occurs
via the fallopian tubes to the ovaries and peritoneal cavity.
The lungs are the most common sites for distant haematogenous metastasis. Non-endometrioid tumours have a tendency for
early dissemination. Even minimal myometrial invasion in these
tumours may be associated with extra-uterine disease.

Wendy MacNab
Mohamed K Mehasseb

Abstract
Endometrial cancer is the most common gynaecological malignancy,
and an understanding of its presentation and management options
is required for all gynaecologists. The surgical management of endometrial cancer has expanded to include laparoscopic surgery, and
debate is ongoing regarding the merits of pelvic and para-aortic lymphadenectomy. Our understanding of the biology of endometrial cancer may help determine which women may benet most from chemoor hormonal therapy in an adjuvant setting. Pelvic radiation is associated with improved local control. Combination therapy with radiation
and chemotherapy is under evaluation. The use of sentinel lymph
node sampling is an area for further research and development.

Keywords chemotherapy; endometrial carcinoma; lymphadenectomy; obesity; radiotherapy

Introduction
Worldwide, endometrial carcinoma is only second to cervical
cancer in frequency among female genital tract cancers. In the
Western world, it is the commonest female genital tract malignancy, accounting for nearly 50% of all new gynaecologic cancers.
Its incidence is rising due to increased life expectancy, obesity
epidemic, and the fewer hysterectomies performed for benign
diseases. Most cases (95%) occur in women over 40 years of age,
mostly in the sixth and seventh decades of life (75e85%). The
overall lifetime risk of developing endometrial carcinoma is 2.5%.
Endometrial carcinoma is usually confined to the uterus at the time
of diagnosis and as such, carries an excellent prognosis with high
curability. However, women with high-risk factors (25%)
including increased age, co-morbidities, higher tumour grade,
aggressive histology, and advanced stage represent real challenges.

Risk factors
i. Obesity
Obesity accounts for about 40%e50% of endometrial cancer
cases in the developed world. Endometrial carcinoma was the
first malignancy to be recognized as being linked to obesity. A
linear increase in the risk of type 1 endometrial cancer with
increasing weight and BMI has been observed (Table 1). Overweight and obese women have two to four times greater risk of
developing endometrial cancer than do women of a healthy
weight, regardless of their menopausal status. Obesity affects the
production of peptides (e.g. insulin and IGF-1, SHBG) and steroid
hormones (i.e. oestrogen, progesterone, and androgens). It is
likely that prolonged exposure to high levels of oestrogen and
insulin associated with obesity may contribute to the development of endometrial cancer.
Obesity in the menopause produces a state of excess oestrogen
production. This is due to the peripheral conversion of androgens
produced by the adrenal glands and ovaries into oestrone, by the
enzyme aromatase, in the adipose tissue. Prolonged unopposed
oestrogen exposure will lead to a continuous spectrum of endometrial changes from proliferative endometrium through hyperplasia/polyps to carcinoma (Figures 1 and 2). Avoiding weight
gain lowers the risk of endometrial and postmenopausal breast

Pathology
Histologically and biologically, endometrial cancer is broadly
classified into two main categories: type 1 and type 2. The vast
majority (80%) of endometrial malignancies are type 1 i.e. endometrioid adenocarcinoma, arising from the glandular epithelium,
usually on a background of atypical hyperplasia. Endometrial
adenocarcinoma is found in up to 50% of cases of complex atypical hyperplasia. Endometrioid tumours are assigned a grade (1
e3) depending on the degree of differentiation and nuclear

Wendy MacNab MBChB MRCOG is a Subspecialty Trainee in


Gynaecological Oncology at Glasgow Royal Inrmary, Glasgow,
Scotland, UK. Conicts of interest: none declared.
Mohamed K Mehasseb MBBCh MSc MD MRCOG PhD is a Consultant
Gynaecological Surgeon and Oncologist at Department of
Gynaecological Oncology, Glasgow Royal Inrmary, Glasgow,
Scotland, UK. Conicts of interest: none declared.

OBSTETRICS, GYNAECOLOGY AND REPRODUCTIVE MEDICINE 26:7

193

2016 Elsevier Ltd. All rights reserved.

REVIEW

mass screening of the population for endometrial cancer with


pelvic ultrasound scans or endometrial biopsies is not practical
due to the low prevalence of the precursor disease. Thus, management relies on the prompt assessment of symptomatic
women, especially those at high risk. It is appropriate to evaluate
individuals past their fourth decade of life if there is abnormal
bleeding.

Relative risk of endometrial cancer incidence and


mortality in relation to body mass index (BMI)
Body mass index

Incidence
Mortality

25e27.4

27.5e29.5

>30

Overall trend for


overweight and obese

21%
9%

43%
21%

273%
228%

289%
246%

Diagnosis
Endometrial cancer most commonly presents as postmenopausal bleeding (PMB) (90%), although only 10% of
women with PMB will have cancer. Other women can present
with persistent postmenopausal vaginal discharge due to pyometra. Premenopausal women usually present with significant
worsening in menstrual pattern, or with incidental finding of
abnormal endometrial cells on routine cervical cytology. Malignant endometrial cells appear on cervical cytology screening
smears in 25e50% of women with endometrial cancer. The
significance of normal endometrial cells in cervical smears in
postmenopausal women is less clear. Presentation as a result of
metastatic disease is uncommon and pain is generally not a
feature.
Prompt referral and initial assessment should take place in
rapid-access clinics, identifying risk factors and comorbidities. A pelvic examination should be conducted to
exclude obvious lower genital tracts cancers. A transvaginal
ultrasound scan is recommended to measure the endometrial
thickness and identify any ovarian mass. A thin endometrium
(<5 mm) in the postmenopausal woman has a high negative
predictive value for endometrial cancer and is reassuring. Ultrasound is less helpful in women taking tamoxifen because
typical morphological changes seen with tamoxifen use often
result in false positive ultrasound findings. Hysteroscopy and
endometrial sampling can be performed safely in the outpatient setting in >80% of women, providing prompt reassurance and a diagnosis in those cases where an endometrial
abnormality is suggested on ultrasound scan. The pipelle is the
best endometrial sampling device, with detection rates for
endometrial cancer in postmenopausal and premenopausal
women of 99.6% and 91%, respectively. The sensitivity for
the detection of endometrial hyperplasia is 81%, with a
specificity of 98%.

Table 1

cancers. However, there is limited evidence that intentional


weight loss will lower cancer risk.
Obese women have a poorer outcome compared to lean
women, probably reflecting a true biological effect of adiposity
on survival, a delayed diagnosis in heavier women, and a higher
rate of treatment-associated complications.
ii. Tamoxifen
Tamoxifen, used as an adjuvant treatment in breast cancer, is
associated with a significantly increased (2e5 fold) incidence of
endometrial pathology including endometrial cancers. Both
endometrioid and non-endometrioid endometrial cancers can
develop. There is no evidence to support routine endometrial
screening for asymptomatic women using tamoxifen, although
abnormal genital tract bleeding should be investigated promptly.
This might become less of an issue over the next few years with a
move to the use of aromatase inhibitors as a substitute for
tamoxifen in the adjuvant treatment of breast cancer.
iii. Hereditary
Less than 5% of all endometrial cancers are hereditary.
Compared to the expected rate of endometrial cancer in a general
population, BRCA carriers who did not receive tamoxifen do not
have a significant increase in risk of developing endometrial
cancer. Thus, it seems that screening for endometrial cancer is
not warranted in known BRCA1 or BRCA2 mutations carriers.
Conversely, endometrial cancer is one of the extra-colonic
cancers caused by hereditary non-polyposis colon cancer syndrome (HNPCC) or Lynch II syndrome. This is an autosomaldominant cancer susceptibility syndrome resulting of a germcell line mutation in one of the DNA mismatch repair genes
(MSH2, MLH1, or MSH6). Despite the name of the syndrome,
50% of affected women will develop endometrial carcinoma as
their index malignancy (rather than bowel cancer). Women with
confirmed HNPCC have a 40e60% lifetime risk of developing
endometrial cancer and a 10% risk of developing a number of
other cancers. Strict criteria have been developed to identify
these women at risk (the Amsterdam criteria). There is no uniform screening strategy, and risk-reducing hysterectomy and
bilateral salpingo-oophorectomy are recommended for those
women who have completed their family. Endometrial surveillance with annual endometrial imaging and biopsy is offered to
women with HNPCC who wish to retain their uterus although
this is not proven to be effective in prevention.

Investigations
Once a diagnosis of endometrial cancer has been made, discussion at a recognized specialist gynaecological cancer MDT should
take place. A blood count, renal biochemistry and liver function
tests are performed and further imaging is undertaken to identify
metastatic disease and aid treatment decisions. A chest X-ray is
done as a minimum to identify lung metastases. In some cases
where the risk of lung metastases is higher e.g. carcinosarcoma,
computed tomography scanning (CT) of the thorax may be used
instead. CT may also be helpful in assessing suspected upper
abdominal metastatic disease. Magnetic resonance imaging
(MRI) is used to assess the depth of myometrial invasion and to
identify extension into cervical stroma. MRI is sensitive for this
purpose, accurately predicting depth of invasion and cervical
extension in 92% of cases.

Screening
Although many endometrial cancers develop by way of a precursor lesion (i.e. atypical endometrial hyperplasia), routine

OBSTETRICS, GYNAECOLOGY AND REPRODUCTIVE MEDICINE 26:7

194

2016 Elsevier Ltd. All rights reserved.

REVIEW

cytologic evaluation, total extra-fascial hysterectomy with bilateral salpingo-oophorectomy, and appropriate surgical staging in
women considered at risk for extra-uterine disease. Many would
now consider laparoscopic hysterectomy, including total laparoscopic hysterectomy (TLH) and laparoscopically assisted
vaginal hysterectomy (LAVH) to be standard practice, although
careful patient selection is required. Post-operatively, many
women will require some type of adjuvant therapy to help prevent vaginal vault recurrence and to address disease in lymph
nodes.

Surgical considerations
The obese patient
An elevated BMI increases the risk of perioperative complications and mortality, particularly in the presence of comorbidities such as diabetes, hypertension, coronary artery
disease, sleep apnoea and joint problems. The risks are further
increased with morbid obesity (BMI >40). Obese women
should receive careful counselling about the increased risk of
complications and technical difficulties that may be encountered during surgery. They also require a thorough preoperative
evaluation including cardiovascular and respiratory assessment. Assistance with proper dosing and monitoring of medications should be considered. During surgery, control of the
airway is critical in obese patients and can be even more
difficult in laparoscopic surgery. Venous access can be problematic. Manual handling of obese women can be challenging,
and special hospital beds and operating tables should be
available, to help reduce the risks to staff and patient. Obese
women are at a much higher risk for postoperative complications compounded by associated co-morbidities. Thus, respiratory or cardiac failure, venous thromboembolism, aspiration,
wound infection and dehiscence are all more common in obese
women. Planned admission to high-dependency units, to optimize the physiological reserves, is often advisable. In obese
patients laparoscopic hysterectomy is optimal as it results in
reduced post-operative complications and a shorter hospital
stay compared to open/abdominal surgery.
The abdominal wall anatomy is often distorted by the overhanging skin and fat (panniculus). Obesity is recognized as a
potential limiting factor in the application of laparoscopic surgery
because of a higher rate of failed entry, hindered manipulation
and poor views. Obesity may not allow steep Trendelenburg
because of unacceptably high peak inspiratory pressure. In
addition, obesity may prevent adequate mobilization of the small
bowel out of the pelvis to allow for proper pelvic visualization.
Nevertheless, laparoscopic surgery has additional benefits for the
obese: they have less postoperative ileus, fewer wound infections
and they mobilize more quickly than those undergoing laparotomy. Obesity presents problems with laparotomy incision
placement and closure. Adequate wound antisepsis is necessary,
as obese women are at increased risk of wound infection and
wound failure. Possible aetiologies include decreased oxygen
tension, immune impairment, and tension or secondary
ischaemia along suture lines. Access to the pelvis can be challenging and there is a higher incidence of intraoperative complications due to problems with access or distorted anatomy.
Difficulty with haemostasis, particularly when removing the

Figure 1 Hysteroscopy showing an endometrial polyp associated with


a thickened malignant endometrium.

Figure 2 A surgical specimen following a total abdominal hysterectomy and bilateral salpingo-oophorectomy performed for the treatment of high-grade endometrial carcinoma. Note the presence of a
bulky enlarged uterus with endometrial cancer lling and distending
both cornua and the right fallopian tube.

Staging
The staging system used for endometrial carcinoma is the International Federation of Gynaecology and Obstetrics (FIGO)
classification, revised in 2009. The staging is based upon findings
at surgery and histological assessment of the surgical specimen,
and provides prognostic information (Tables 2 and 3).
After omission from the 2009 FIGO staging for endometrial
carcinoma, the need for peritoneal cytology and the significance of a
positive result became controversial. In women with stage I and II
disease, positive peritoneal cytology results do not influence survival. Poor prognosis associated with positive washings is most
common in women with other adverse prognostic factors: i.e. grade
3 histologic types, metastases to the adnexa, deep myometrial invasion, or positive pelvic or para-aortic nodes. Positive peritoneal
cytology may thus carry a prognostic significance only when the
endometrial carcinoma has spread beyond the uterus.

Management
The standard treatment for endometrial carcinoma remains surgical and comprises of collection of peritoneal fluid washings for

OBSTETRICS, GYNAECOLOGY AND REPRODUCTIVE MEDICINE 26:7

195

2016 Elsevier Ltd. All rights reserved.

REVIEW

Chemotherapy
The value of adjuvant systemic chemotherapy in women with
high-risk early stage endometrial cancer is still controversial.
PORTEC-3 study is a phase III randomized trial comparing chemoradiation and adjuvant chemotherapy (four cycles of carboplatin and paclitaxel) versus pelvic radiation alone in high-risk
and advanced stage disease. This study should determine
whether radiotherapy or chemotherapy improves overall survival and failure-free survival. Two additional GOG studies are
examining the role of adjuvant chemotherapy in the treatment of
endometrial carcinoma (GOG 249 and GOG 258).

FIGO endometrial cancer staging 2009


Stage1
1A
1B

Tumour confined to corpus uteri


No or less than half myometrial invasion
Invasion equal to or more than half the
myometrium
Tumour invades cervical stroma, but not does
not extend beyond the uterusa
Local and/or regional spread of the tumour
Tumour invades the serosa of the corpus uteri
and/or adnexa
Vaginal and/or parametrial involvement
Metastases tp pelvic and/or para-aortic lymph
nodes
Positive pelvic nodes
Positive para-aortic lymph nodes with or
without positive pelvic lymph nodes
Tumour invades bladder and/or bowel
mucosa, and/or distant metastases
Tumour invades bladder and/or bowel mucosa
Distant metastases, including intra-abdominal
metastases and/or inguinal lymph nodes

Stage II
Stage III
IIIA
IIIB
IIIC
IIIC1
IIIC2
Stage IV
IVA
IVB

Controversies in the management of endometrial cancer


Surgical staging and lymphadenectomy
With the increasing emphasis on surgico-pathologic staging, the
need for pelvic and para-aortic lymphadenectomy to accurately
identify lymphatic spread remains a source of controversy. It is
unclear how this information affects prognosis and whether it
can guide the use of adjuvant therapies. The major questions
regarding lymphadenectomy therefore include the need for
routine systemic lymphadenectomy, extent of lymph node
dissection required and whether it incurs any therapeutic
benefit. However, the majority of women with endometrial
carcinoma are low-risk for nodal disease at presentation and
adjuvant treatment decisions can be based on final pathologic
information.

Grading is reported in addition to stage: either G1, G2 or G3.


Positive cytology to be reported separately without changing the stage.
a
Endocervical glandular involvement only should be considered as stage I
and no longer as stage II.

Table 2

The need for routine systemic lymphadenectomy


The strongest argument for routine staging is the avoidance of
pelvic radiation therapy if nodal assessment confirms nodenegative disease and hence low-risk status. In the absence of
nodal disease, recurrence risk is low and overall survival is high
with no radiation or with the substitution of external beam
radiotherapy with vaginal vault brachytherapy.
Other arguments for routine staging include the inaccuracy of
preoperative or intraoperative assessments of nodal involvement
(up to 10% of women with apparently low risk disease have
lymph node spread) and the lack of significant morbidity associated with the procedure. This has encouraged many gynaecologic oncologists to move towards performing routine surgical
staging including pelvic and para-aortic lymphadenectomy for
most women with endometrial cancer.
However, a randomized controlled trial in which women with
stage I endometrial cancer were assigned to have a standard
hysterectomy and ovary removal with or without lymphadenectomy, found that systematic use of pelvic lymphadenectomy
did not improve disease-free or overall survival in women with
early-stage endometrial cancer. It should be recognized that the
major complication rate of pelvic lymphadenectomy is approximately 2e6%, which may be an unacceptable risk in women
who are highly likely to have early stage disease.
It could be argued that the increased use of radiation in
unstaged women may produce similar outcomes to women who
are staged and who avoid radiation therapy. This was addressed
by the randomized trial A Study in the Treatment of Endometrial
Cancer (ASTEC) that randomized women with endometrial
cancer treated with hysterectomy to pelvic lymphadenectomy or
not. Following surgery, women with stage I-IIA disease were

cervix and suturing the vaginal vault, requires experience to


manage. Good assistance, retraction and lighting are essential.
Radiotherapy
Pelvic radiotherapy (external beam radiotherapy (EBRT) or
vaginal brachytherapy (VBT)), is crucial in the management of
endometrial cancer, whether used as an adjuvant treatment after
surgery or as definitive treatment for women who are medically
inoperable or with local recurrence. The need for post-operative
radiotherapy is usually determined by prognostic features obtained from the pathology review. Radiation therapy decreases
the risk of pelvic recurrence. However, adjuvant pelvic radiotherapy leads to improved local control but no overall survival
advantage in women with low-risk endometrial cancer.
According to the PORTEC-2 study, VBT is considered a more
convenient mode of treatment with less morbidity, providing
optimal local control and preventing vaginal recurrence with best
quality of life compared to EBRT.

FIGO endometrial cancer grading system


Differentiation Grade Description
Well
Moderate
Poor

1
2
3

5% of glandular component is solid


6e50% of the glandular component is solid
>50% of glandular component is solid

Table 3

OBSTETRICS, GYNAECOLOGY AND REPRODUCTIVE MEDICINE 26:7

196

2016 Elsevier Ltd. All rights reserved.

REVIEW

not, making interpretation of results difficult. The number of


lymph nodes resected was insufficient in many women, and the
inclusion of a large number of low-risk women may account for
the low rate of positive lymph node involvement detected in the
ASTEC trial.

then randomized again to observation or pelvic radiation therapy


if they had grade 3, serous, or clear cell histology; >50% myometrial invasion; or endocervical glandular invasion (i.e. high
risk disease). The results suggested that there was no advantage
for routine systematic lymphadenectomy.
An alternative approach is to reserve nodal dissection for
women with high risk of nodal disease. It seems that the depth of
myometrial invasion is the most important factor that determines
the likelihood of nodal involvement. According to the GOG 33
study, the risk of pelvic nodal disease is around 3% for all
women with grade 1 tumours, increasing to 11% with deeply
invasive (outer one-third myometrial invasion) tumours. Meanwhile, women with grade 3 tumours had a risk of pelvic nodal
metastases of circa 18%, and 34% with deep invasion. Women
with serous or clear cell histology also warrant nodal dissection,
as 30%e50% will have nodal disease.
It is suggested that lymphadenectomy improves the
carcinoma-related survival and the recurrence free survival in
high-risk endometrioid adenocarcinoma women. Conversely,
lymphadenectomy does not appear to benefit women with grade
1 and 2 endometrioid lesions with myometrial invasion <50%
and primary tumour diameter <2 cm. There is increasing evidence that systematic lymphadenectomy in low-risk cases (grade
1 or 2 endometrioid tumours confined to the inner half of the
myometrium) is unnecessary.

Predicting nodal disease


In an effort to minimize the impact of lymphadenectomy and
assist selective lymph node dissection, predicting nodal disease
using positron emission tomography (PET) has been attempted.
18F-FDG PET/CT has been demonstrated to be an accurate
method for the pre-surgical evaluation of pelvic nodes metastases
with high sensitivity, specificity and positive predictive value. In
an attempt to avoid complete lymphadenectomy, the concept of
sentinel node identification has been investigated in endometrial
carcinoma. A recent trial e SENTI-ENDO has found that the use
of sentinel lymph node sampling is feasible with a high negative
predictive value (NPV). In women with positive sentinel node(s),
knowing its exact location from sampling can be used to tailor
adjuvant treatment. This is an area for further research and
development.

Advanced and recurrent disease


Women with advanced or recurrent disease need careful individual consideration in order to define the goals of treatment
and balance these against performance status and quality of
life. Where disease is advanced at first presentation, it is
generally preferable to excise the uterus prior to radiotherapy
or chemotherapy particularly where there is troublesome
vaginal bleeding and pelvic pain. Where disease involves the
lower vagina at presentation, treatment is usually primary
radiotherapy. Where advanced (stage III or IV) disease is
diagnosed following surgery, multimodal treatment is used.
Adjuvant external beam radiotherapy to the pelvis and vaginal
vault brachytherapy are applied to treat residual disease and
prevent local recurrence, with the addition of adjuvant
chemotherapy to prevent distant disease. Treatment for
advanced disease with widespread nodal involvement at presentation is usually palliative and the sequence of treatments
may be determined by the most troublesome symptoms. For
example, chemotherapy may be given to treat systemic disease
initially and external beam radiotherapy or surgery can subsequently be used, if necessary, to palliate vaginal bleeding.
Treatment with high dose oral progestogens is useful for
palliation in advanced disease and/or where the patient is unfit
for other types of treatment. Good symptomatic relief can be
achieved and tumour deposits may regress over a number of
months although high-grade tumours frequently lack progesterone receptors and may therefore respond less readily. Side
effects of hormonal therapy include weight gain, venous
thromboembolism and hypertension.
Women who experience recurrence after treatment for early
endometrial cancer are managed according to the pattern of
recurrence and overall fitness. MRI is useful for initial evaluation of suspected pelvic recurrence, although CT is required to
assess the abdomen and thorax for other metastases in this

Extent of lymphadenectomy
When full surgical staging is performed, a bilateral pelvic and
para-aortic lymphadenectomy is increasingly advocated
because positive lymph nodes (including isolated para-aortic
lymph nodes) can occur in all grades. The clinico-pathologic
factor most strongly related to para-aortic nodal metastasis is
pelvic lymph node metastasis, hence, systemic para-aortic
lymphadenectomy is advocated on all high-risk women, or in
women with two or more positive pelvic lymph nodes. However, this is a major surgery to undertake in women who are
usually elderly and obese, with other co-morbidities. An
extensive para-aortic lymphadenectomy significantly increases
operating time and blood loss and also increases postoperative
morbidity, particularly lower limb lymphoedema (in about 20%
of women). It could thus be argued that primary prevention of
lymphoedema by selective use of pelvic lymphadenectomy and
avoidance of systematic para-aortic lymphadenectomy is highly
desirable.
Therapeutic role of lymphadenectomy
The therapeutic role of lymphadenectomy is less well defined.
The current available evidence from the ASTEC trial does not
support the claim that lymphadenectomy in endometrial cancer
is therapeutic. Unfortunately, there are several reasons why the
ASTEC trial could have failed to show improved overall survival with routine lymphadenectomy. The trial required only a
pelvic lymphadenectomy and utilized a second randomization
for pelvic radiation for disease characteristics, which, following
a negative nodal dissection, is typically avoided. Likewise,
vaginal vault radiation was permitted as per institutional
practice irrespective of the assignment to pelvic radiation or

OBSTETRICS, GYNAECOLOGY AND REPRODUCTIVE MEDICINE 26:7

197

2016 Elsevier Ltd. All rights reserved.

REVIEW

performed. Recurrence may be suggested by vaginal bleeding,


new onset of persistent backache, significant weight loss or
persistent pressure symptoms. The frequency and duration of
follow-up visits vary between hospitals and usually follow
guidelines developed locally within each cancer network. The
role of the routine follow-up visit in detecting asymptomatic
recurrence and improving survival from recurrence is unproven.
It is also recognized that for some women, routine follow-up after
a cancer diagnosis may be a stressful experience. The true value
of routine hospital-based follow-up visits has yet to be properly
determined.

setting. Isolated vaginal vault recurrence can be successfully


salvaged with either surgery or radiotherapy with 3-year survival rates of approximately 70%. Other pelvic recurrences are
usually treated with radiotherapy. Distant metastases can be
treated with chemotherapy, as discussed above, or high dose
progestogens. Radiotherapy can also be used with good effect to
treat isolated bony metastases, which can cause pain and
disability.

Endometrial carcinomas in young women


Less than 5% of endometrial cancers occur in women aged 40
years or younger, and the majority are well-differentiated tumours (90% of lesions), limited to the endometrium. A more
variable histologic pattern may occur in association with Lynch
syndrome (HNPCC). Fertility preservation could be an issue in
these women and progestogen therapy may be used. Endometrial
carcinoma has been reported to progress in 80% of cases with
various progestational agents (e.g. megestrol acetate 160e320
mg/day or medroxyprogesterone acetate 200e500 mg/day),
however, successful pregnancies have been reported in only 40%
of these women. A decision for uterine preserving treatment
should not be taken lightly. It is crucial to exclude the presence of
significant myometrial invasion by MRI, and to ascertain a grade
1 histology and progesterone receptor positive status. Ovarian
pathology has to be excluded in particular in association with
Lynch syndrome, and synchronous primary or metastatic disease
ruled out.
The endometrial response needs to be assessed after 3 months
of progestogen treatment. Unfortunately, about 40% of women
who initially respond will recur, hence the need for prolonged
surveillance and the recommendation for hysterectomy once
childbearing is completed.
Following hysterectomy and bilateral salpingo-oophorectomy,
pre-menopausal women with endometrial cancer will experience
menopausal symptoms, which can be severe for some women
causing distress and impairing quality of life. Women experiencing significant symptoms may be prescribed HRT after
receiving appropriate information about risks and benefits. An
alternative treatment is the anti-depressant drug, venlafaxine,
which can be very effective in reducing vasomotor symptoms.
HRT should be avoided in women with more advanced disease
or metastatic disease.

Future directions
Advances in the understanding of the molecular basis of endometrial cancer may enable future development of more targeted
therapies.
PTEN is a regulatory protein, expressed most highly in oestrogen rich environments, and is involved in the regulation of
cellular growth, proliferation, survival and protein transcription. Mutations within PTEN are observed in many type 1
endometrial cancers, resulting in hyperactivation of mTOR
(mammalian target of rapamycin) and thus, inhibition of mTOR
and/or PI3K has been proposed as a potential treatment target.
As such mTOR inhibitors have been developed and demonstrated promising early results. Temsirolimus showed a 26%
response rate and 63% disease stabilization rate in a phase II
trial of chemotherapy nave women. These initial results have
prompted further trials in combination with established treatment regimens.
Inhibition of angiogenesis has also been proposed as a
mechanism for more targeted therapies. Correlations between
the expression of vascular endothelial growth factor (VEGF) and
prognostic factors in endometrial cancer have been observed.
Bevacizumab is a recombinant monoclonal antibody directed
against VEGF and several phase II trials are currently examining
its efficacy.
Epidermal growth factor (EGFR) is involved in the promotion
of cell proliferation and its overexpression has been implicated in
the development of a number of tumours. EGFR inhibitors have
had excellent effect in the management of both breast and colon
cancer. Whilst EGFR overexpression has been demonstrated in
both type 1 and type 2 endometrial cancers, the results of EGFR
inhibitor trials have been disappointing.
A

Prognosis and follow up


The overall prognosis for endometrial cancer is generally good
and reflects early presentation of the disease in most cases. The
5-year survival rate for all stages is approximately 80% but varies
with tumour grade and depth of myometrial invasion. Survival in
stage 1 disease is 85e90% but approximately 70e75% for stage
II, 45% for stage III and <30% for stage IV disease. Other factors
that adversely affect prognosis include non-endometrioid histological sub-type and lymphovascular space invasion.
Most endometrial cancer recurrences happen within the first 3
years after treatment. Follow-up is undertaken with the aim of
detecting recurrence and identifying side-effects of treatment. A
history of symptoms is taken and then clinical examination
(including pelvic vaginal and rectal examination) is usually

OBSTETRICS, GYNAECOLOGY AND REPRODUCTIVE MEDICINE 26:7

FURTHER READING
ASTEC Study Group. Efcacy of systematic pelvic lymphadenectomy
in endometrial cancer (MRC ASTEC trial): a randomised study.
Lancet 2009; 373: 125e36.
Ballester M, Dubernard G, Lecuru F, et al. Detection rate and diagnostic accuracy of sentinel-node biopsy in early stage endometrial
cancer: a prospective multicentre study (SENTI-ENDO). Lancet
Oncol 2011; 12: 469e76.
Colombo N, Creutzberg C, Amant F, et al. on behalf of the
ESMO_ESGO_ESTRO Endometrial Consensus Conference
Working Group. ESMO-ESGO-ESTRO consensus conference
on endometrial cancer: diagnosis, treatment and follow-up. Ann
Oncol 2015; 1e26. http://dx.doi.org/10.1093/annonc/mdv484.

198

2016 Elsevier Ltd. All rights reserved.

REVIEW

Humber CE, Tierney J, Symonds PR, et al. Chemotherapy for


advanced, recurrent or metastatic endometrial carcinoma.
Cochrane Database Syst Rev 2005; 4.
Intercollegiate Guidelines Network. Investigation of postmenopausal
bleeding. Guideline 61. Edinburgh: SIGN, 2002.
Kong A, Johnson N, Cornes P, et al. Adjuvant radiotherapy for stage I
endometrial cancer. Cochrane Database Syst Rev 2007; 18:
CD003916.
National Institute for Health and Clinical Excellence. Laparoscopic hysterectomy (including laparoscopically assisted
vaginal hysterectomy) for endometrial cancer. London: NICE,
2010.
Nout RA, Putter H, Jurgenliemk-Schulz IM, et al. Vaginal brachytherapy versus external beam pelvic radiotherapy for high-intermediate
risk endometrial cancer: results of a randomized PORTEC-2 trial):
an open-label, non-inferiority, randomised trial. Lancet 2010 Mar 6;
375: 816e23.
Walker JL, Piedmonte MR, Spirtos NM, et al. Laparoscopy compared
with laparotomy for comprehensive surgical staging of uterine
cancer: Gynecologic Oncology Group Study LAP2. J Clin Oncol
2009; 27: 5331e6.

OBSTETRICS, GYNAECOLOGY AND REPRODUCTIVE MEDICINE 26:7

Practice points
C

199

Early stage endometrial cancer carries an excellent prognosis and


prompt recognition and investigation of symptoms is therefore vital
Prognostic factors in endometrial cancer include tumour grade,
depth of myometrial invasion, non-endometrioid histological
subtype and lymphovascular space invasion
Surgery remains the foundation of treatment and current evidence
supports the safety and efficacy of minimally invasive surgery
Adjuvant radiotherapy reduces the risk of local recurrence but
provides no overall survival advantage
The role of chemotherapy in women with high risk early stage
disease remains unclear and the results of several ongoing trials
are awaited. In women with advanced or recurrent disease,
chemotherapy improves disease free survival with a modest
improvement in overall survival
The role and extent of lymphadenectomy is controversial. Randomised controlled trials have failed to demonstrate survival
advantage in clinical stage 1 disease. The use of sentinel lymph
node sampling an area for further research and development

2016 Elsevier Ltd. All rights reserved.

REVIEW

In vitro fertilization

Indications
Since the birth of Louise Brown in 1978, a significant number of
developments in both clinical medicine and basic sciences have
given a chance to an increasing number of infertile couples to
have a baby. IVF was initially developed for women with tubal
disease, but its indications have now expanded significantly with
the changing socio-economic dynamics of our society. One
million babies were born in the first 25 years of IVF between
1978 and 2003 and it has been estimated that five million more
children have been born by the end of 2013. In the United
Kingdom (UK) 224,196 babies were born after IVF treatment
between 1991 and 2011. Table 1 provides a list of currently
accepted indications for ART.
Couples attending for their initial consultation before
embarking on IVF treatment should be given lifestyle advice
(Figure 1), offered general screening and specific infertility
investigation work-up.

Mythili Ramalingam
Punukollu Durgadevi
Tahir Mahmood

Abstract
In vitro fertilization, popularly referred to as IVF, changed the infertility
practice worldwide since its introduction in 1978 and it has continued
to evolve. Assisted reproductive technology (ART) refers to all treatments or procedures that include the in vitro handling of both
human oocytes and sperm, or embryos, for the purpose of establishing a pregnancy. Today ART is available globally, and the current practice is largely different from that which was used during the early days
of its inception. Although advances in laboratory technology and clinical practice have allowed IVF to evolve, it still remains invasive,
expensive and is associated with risks. This article reviews the current
indications for assisted reproduction and the preparatory steps before
starting treatment. It also summarizes the treatment cycle including its
main clinical and laboratory aspects, and the complications associated with treatment.

General infertility work-up


The basic aim of the investigations is to establish if ovulation
occurs (by mid-luteal progesterone estimation), fallopian tubes
are patent and the male partner has normal semen analysis.
Rubella immunity should be checked to allow immunization
for those females who are non-immune, before embarking on IVF
treatment.
Chlamydia screening should be carried out before organizing
tests for tubal patency. Tubal patency testing is carried out either
by hysterosalpingogram (HSG) or laparoscopy and dye test. HSG
is a preferable first line investigation for women with no previous
history of pelvic inflammatory disease, chlamydia exposure or
other risk factors for tubal disease.
An ultrasound scan of the pelvis is performed to rule out
congenital abnormalities of the uterus, hydrosalpinges, endometrial polyps, fibroids or ovarian endometriomas (Figure 2).
An up to date cervical screening test should be available and
referral to colposcopy be organized if indicated.
A semen analysis should be performed for all men attending
the infertility clinic. Cystic fibrosis screening is performed in
patients with family history of the disease or couples in whom
male partner has obstructive azoospermia with congenital bilateral absence of the vas deferens or totally immotile sperms
(immotile cilia syndrome).
Screening for haemoglobinopathies (sickle cell disease and
thalassaemia) should be offered in the high-risk population.

Keywords assisted reproduction; IVF; ovarian hyperstimulation


syndrome

Introduction
It is estimated that infertility affects 1 in 7 heterosexual couples in
the UK. An increasing number of couples are seeking ART to
conceive and ultimately giving birth to a healthy live baby. ART
has opened many avenues for the infertile couples, albeit with a
financial and inherent psychological cost. The Warnock Report
(1984) set out the principles to safeguard the developments in
medicine and science related to human fertilization and embryology. This was followed by the creation of Human Fertilisation
and Embryology Act of 1990, and the establishment of a regulatory body The Human Fertilisation and Embryology Authority
(HFEA) in the United Kingdom.
IVF involves several steps: initial assessment and counselling,
suppressing the natural cycle, ovarian stimulation and monitoring, oocyte retrieval, fertilization of the eggs, embryo transfer
and luteal support, pregnancy test and confirmation of viability
of pregnancy with ultrasound.

Criteria for referral/funding for IVF


The recommendations of the National Institute for Health and
Care Excellence (NICE) guidelines are show in Box 1.
Mythili Ramalingam MRCOG is an Academic Clinical Lecturer in
Reproductive Medicine at the University of Dundee and Ninewells
Hospital, Dundee, UK. Conicts of interest: none declared.

Investigations prior to IVF


The specific investigations for patients before they embark on
treatment for IVF include:
 Screening for blood borne viruses (BBV) specifically HIV
and Hepatitis B and C infection, to reduce risk of transmission of infection to the unaffected partner and offspring
and also to reduce the theoretical risk of cross infection
and contamination among cryopreserved embryos.

Punukollu Durgadevi MRCOG is a Consultant Obstetrician and


Gynaecologist at Victoria Hospital, Kirkcaldy, Fife, UK. Conicts of
interest: none declared.
Tahir Mahmood MD FRCPE MBA FACOG FRCOG is a Consultant
Obstetrician and Gynaecologist at Victoria Hospital, Kirkcaldy, Fife,
UK. Conicts of interest: none declared.

OBSTETRICS, GYNAECOLOGY AND REPRODUCTIVE MEDICINE 26:7

200

2016 Elsevier Ltd. All rights reserved.

REVIEW

The HFEA provides clinics with consent forms for different


treatment options:
 Consent to the fertility treatment
 Disclosure of information and
 The use and storage of gametes and/or any embryos produced from them
These forms are ideally completed before starting the treatment process. IVF treatment should not be offered if there is a
concern regarding the welfare of the child (e.g. history of drug or
child abuse). Decisions about storage and posthumous use of
gametes and embryos should be discussed and the outcome of
the discussion recorded in the case notes.

The indications for assisted reproduction


Indications
Tubal

Severe bilateral tubal (proximal or distal or


both) disease not amenable to surgery or after
failed tubal surgery or bilateral
salpingectomies
Endometriosis
if moderate or severe disease when
conception has not occurred within 12 months
of ablative surgery
Anovulation
Type I (Hypogonadotropic hypogonadal
anovulation) anovulatory patients after failed
gonadotropin releasing hormone agonist
(GnRH agonist) pump or gonadotropins
Type II (Hyperandrogenic anovulation)
anovulatory patients who are resistant to oral
antiestrogens (clomiphene citrate/letrozole) or
after failure of gonadotropins/ovarian drilling
Type III (Hypergonadotropic hypogonadal
anovulation) anovulatory patients with
diminished ovarian reserve as they have
limited window of opportunity before full
picture of ovarian failure
Unexplained infertility Duration of more than 2 years especially
where less invasive treatments have failed
Male factor
Azoospermia or one or combinations of the
following on more than one sample taken 3
months apart; severe oligozoospermia
(<10.6/ml), asthenozoospermia (<40%
progressive motility or <25% fast forward
progressively motile sperms),
teratozoospermia (<5% of normal sperms
using strict morphology criteria)
Other
C Coital dysfunction, gamete donation, clinical
situation where PGD (preimplantation genetic
diagnosis) or PGS (preimplantation genetic
screening) are required

Pre-treatment interventions
Stress can have impact on the outcome of IVF treatment. It is
therefore important for couples to have access to professional
counsellors before, during and more importantly following failed
treatment. It is debatable if hysteroscopy or hysterosonography
should be done routinely before the start of IVF treatment in
order to improve sensitivity of detection of intrauterine adhesions or polyps that may escape detection by routine pelvic ultrasound. A recent Cochrane review has reported that
hysteroscopic removal of asymptomatic uterine polyp and
hydrosalpinges prior to IVF treatment improves pregnancy rates
significantly. The different types of IVF are summarized in Table
2.

Treatment cycle
The treatment cycle comprises of seven main steps as follows:
1. Pituitary down regulation
2. Ovarian stimulation
3. Monitoring of response to ovarian stimulation
4. Ovulation trigger
5. Oocyte collection
6. Luteal support
7. Embryo transfer
Pituitary down regulation (suppression)
During a natural cycle, there is a natural surge of LH in the midluteal phase before ovulation. The main aim of pituitary down
regulation is to avoid the effect of the intrinsic hormones on the
ovaries during the stimulation process. In addition to avoiding
the premature LH surge it also helps for planned oocyte retrieval
at a time that is convenient for the couples and staff in assisted
conception unit. The GnRH antagonist protocol reduces risk of
ovarian hyper-stimulation by allowing use of GnRHa trigger for
final oocyte maturation instead of Human Chorionic Gonadotropin (HCG). In women with oligo- or amenorrhoea progesterone withdrawal bleed is induced prior to down-regulation.
The most commonly used protocols for pituitary suppression
include long and short protocols (Figure 3). In these protocols
Gonadotropin releasing Hormone agonists (GnRHa) are used for
pituitary suppression. In the long protocol there is an initial
stimulatory effect of GnRHa on the pituitary leading to transient
surge of FSH and LH. This is usually followed by profound pituitary suppression 10e14 days later, leading to hypooestrogenic state amounting to menopausal status. Pituitary
suppression is confirmed by transvaginal measurement of

Table 1

 Ovarian reserve testing is a measure of the quantitative


and qualitative assessment of oocytes in the ovaries of
women. A number of markers of ovarian reserve are used
which includes an estimation of levels of Follicle stimulating hormone (FSH), Anti-Mullerian hormone (AMH)
and inhibin B. AMH is the most commonest type used
because the intercycle and intracycle variation is very
small. Furthermore, ultrasound assessment of antral follicle count (AFC) has also been used to assess the likely
ovarian response to stimulation as this indirectly affects
the outcome of IVF cycle.
 Of all the ovarian reserve tests described, AFC and AMH
are the most sensitive available tests to predict ovarian
response to treatment in ART. These tests help in proper
planning of ovarian stimulation and for counselling of
couples regarding risks of poor response, although both
perform modestly in predicting pregnancy rates.

OBSTETRICS, GYNAECOLOGY AND REPRODUCTIVE MEDICINE 26:7

201

2016 Elsevier Ltd. All rights reserved.

REVIEW

Lifestyle advice
Lifestyle advice

Smoking cessation

Weight

Improves ovarian response


Improves quality of oocytes
Improves obstetric outcome

Stopping alcohol/
recreational drugs

Folic acid

Reduces teratogenic
risk in pregnancy
Take 400 g/day for
3 months prior to conception
5 mg/day (e.g. DM, obese,
previous NTD & thalassaemia)

Aim for BMI < 30/m2


Improves ovarian response
Reduces obesity-associated
risk in pregnancy
Increases success of IVF

Preconception counselling

Epilepsy
Diabetes Mellitus
Others

Figure 1

endometrium (<5 mm). Some units also test Follicle Stimulating


Hormone (FSH), Luteinizing Hormone (LH) and Oestradiol (E2)
levels in addition to endometrial measurements, prior to starting
ovarian stimulation by administering GnRHa by subcutaneous
injection or by daily inhalation from early follicular or mid-luteal

phase of the spontaneous cycle preceding treatment. The short


protocol builds on the early follicular phase to maximize the
ovarian response. The gonadotropins are introduced between
day 3 and 5 of the cycle. The advantage of the short protocol is
that it has shorter duration of treatment compared to the long

Ultrasound images

Intrauterinepolyp

Endometrioma

Hydrosalpinx
Figure 2

OBSTETRICS, GYNAECOLOGY AND REPRODUCTIVE MEDICINE 26:7

202

2016 Elsevier Ltd. All rights reserved.

REVIEW

are measured along with endometrial thickness. The monitoring


typically starts 7e8 days after the confirmation of pituitary down
regulation and the start of gonadotrophins ovarian stimulation.
The TVUSS is carried out on alternate days until the follicles are
mature and ready for ovulation trigger.
The criterion of an adequate response in most centers will be
the availability of at least three leading follicles measuring 17
e18 mm in diameter. In some units ovarian monitoring by
TVUSS is combined with measurements of E2/FSH/LH and
progesterone levels.

Criteria for funding for IVF


C

A full cycle of IVF treatment, with or without intracytoplasmic


sperm injection (ICSI), should comprise one episode of ovarian
stimulation and the transfer of any resultant fresh and frozen
embryo(s).
In women aged less than 40 years who have not conceived after 2
years of regular unprotected intercourse or 12 cycles of artificial
insemination (where six or more are by intrauterine insemination), offer three full cycles of IVF, with or without ICSI.
If the woman reaches the age of 40 during treatment, complete
the current full cycle but do not offer further full cycles.
In women aged 40e42 years who have not conceived after 2
years of regular unprotected intercourse or 12 cycles of artificial
insemination (where six or more are by intrauterine insemination), offer one full cycle of IVF, with or without ICSI, provided the
following three criteria are fulfilled (They have never previously
had IVF treatment, there is no evidence of low ovarian reserve
there has been a discussion of the additional implications of IVF
and pregnancy at this age).
The implementation of guidelines varies throughout the UK and
are based on strict eligibility criteria.

Ovulation trigger
The ovulation trigger for final oocyte maturation is usually achieved by administering either Human chorionic gonadotropin
(HCG) or GnRH agonist in a cetrotide protocol. This is equivalent
of the mid-luteal LH surge that happens in natural cycle. In
addition to this, it also helps in resumption of meiosis in the
oocytes and thereby helping successful fertilization. Therefore it
is important to time ovulation trigger with clear instructions to
the patients.
Oocyte collection
Oocyte retrieval is usually done under sedation/general anaesthesia. TVUSS guided oocyte retrieval is scheduled 35.5 hours
after ovulation trigger. This is performed with a single-lumen 18gauge needle (Figure 5). All the follicles, even those less than 10
mm, are aspirated. The follicular fluid aspirated is received into
pre-warmed test tubes to avoid any wide fluctuation of temperature that can damage the meiotic spindles of the oocytes. This
fluid is then checked promptly by the embryologist in the laboratory to retrieve oocytes and transfer them to the culture medium under appropriate conditions in the incubator.

Box 1

protocol in addition to shorter duration of the gonadotropins and


cost. The disadvantage, however, is that it provides less protection from premature ovulation.
Antagonist protocol
In this protocol GnRH antagonists are used and they bind and
block the GnRH receptors in the pituitary gland. This has a
negative feedback on the pituitary and therefore reduction in
FSH, LH and oestrogen. The antagonist protocol is used in
women at high risk of ovarian hyperstimulation syndrome
(Figure 4).

Luteal support
The luteal phase in IVF cycles is insufficient because of pituitary
suppression. Therefore women are administered luteal phase
support to improve implantation rates and thus pregnancy rates.
A recent Cochrane review showed that the live birth rate was
significantly higher with the progesterone for luteal phase support in IVF/ICSI cycles, favouring synthetic progesterone over
micronized progesterone. The addition of other substances such
as oestrogen or HCG did not seem to improve outcomes. There is
no evidence, thus favouring a specific route or duration of
administration of progesterone.

Ovarian stimulation
Ovarian stimulation is achieved usually by exogenous gonadotropins. There are currently two preparations in the market: the
first one extracted from urine of postmenopausal women (human
menopausal gonadotropins, HMG) and the second one being
recombinant gonadotropins. There is no evidence of difference in
effectiveness between urinary derived HMG and recombinant
FSH preparations in ART outcomes. Poor response is reported in
the range of 10e15% of patients. This is a major cause of cycle
cancellation and can be distressing to couples. In contrast severe
ovarian hyper-stimulation (OHSS) can occur in 0.5e2%of
women. Individualized stimulation protocols for IVF have been
introduced recently. This approach has been shown to be comparable to conventional ovarian stimulation in terms of live birth
rates however with less risk of ovarian hyper-stimulation and
multiple pregnancies.

Laboratory aspects
The oocytes are retrieved from the follicular fluid and transferred
to the culture medium under appropriate conditions in the
incubator. In this process, they are graded for maturity by the
embryologist. In IVF the oocytes are left in a small petri dish
along with prepared spermatozoa. In ICSI, the outer cumulus
cells are stripped from the oocyte before injection with single
sperm. Evidence of fertilization is checked 16e20 hours
following incubation or injection. The embryos are then incubated for a further 2e5 days for further development. The
embryologist grades them according to the number of cells, degree of fragmentation and regularity. The best embryos are
selected for transfer leaving any good quality embryos for
cryopreservation.

Monitoring of response to ovarian stimulation


Transvaginal ultrasound scanning (TVUSS) of the ovaries is the
mainstay of monitoring the ovarian response. During ultra-sound
scanning, the number and size of recruited follicles in each ovary

OBSTETRICS, GYNAECOLOGY AND REPRODUCTIVE MEDICINE 26:7

203

2016 Elsevier Ltd. All rights reserved.

REVIEW

Different types of IVF


Types of ART

Definition

Indication

Success

IVF

IVF treatment involves the


fertilization of an egg (or eggs)
outside the body. This can be
performed with own eggs and
sperm, or donated sperm or
donated eggs, or both

Unexplained infertility, tubal blockage


Unsuccessful IUI, male factor infertility
but not severe enough to require ICSI
and embryo testing

ICSI

Intra-cytoplasmic sperm
injection (ICSI) is where a
single sperm is injected
directly into an egg
Intrauterine insemination
(IUI) is when the fast-moving
sperm is separated from more
sluggish or non-moving sperm
and then inseminated inside
the uterus

Oligoasthenoteratozoospermia, poor
or failed fertilization, surgical sperm
retrieval, using frozen sperm and
embryo testing
Donor insemination, inability to have
vaginal intercourse because of a physical
disability or psychosexual problem,
BBV ve males to perform sperm
washing to reduce the risk
transmission of infection

In 2010 for IVF cycles using fresh embryos


created with their own eggs the success rate
was:
C
32.2% for women aged under 35
C
27.7% for women aged between 35e37
C
20.8% for women aged between 38e39
C
13.6% for women aged between 40e42
C
5.0% for women aged between 43e44
C
1.9% for women aged 45 and over
Intra-cytoplasmic sperm injection (ICSI)
success rates are very similar to IVF rates

DI

Donor insemination (DI) uses


sperm from a donor to help the
woman become pregnant.
This can be used for either
DI e IUI/IVF/ICSI

Male factor infertility

GIFT

Gamete intrafallopian
transfer (GIFT) uses the
healthiest eggs and sperm to
fertilize together in the
womans fallopian tubes
In vitro maturation (IVM) is
a process where the eggs are
removed from the ovaries when
immature. They are then matured
in the laboratory before being
fertilized

Unexplained infertility, patent fallopian


tubes, oligo or asthenozoospermia,
religious objections to IVF and previous
failed IVF

IUI

IVM

Risk of OHSS (ovarian hyperstimulation


syndrome), childhood cancers and
male factor infertility

In the year 2008 for women receiving IUI with


donor sperm the live birth rate was:
C
15.8% (237/1497) for women <35
C
11.0% (154/1394) for women between
35e39
C
4.7% (23/492) for women between 40e42
C
1.2% (2/172) for women aged between
43e44
Figures given in brackets are (IUI cycles
resulting in a live birth/IUI cycles started)
In 2010 for unstimulated DI that resulted in a
live birth was:
C
14.6% for women aged <35
C
11.4% for women aged 35e37
C
9.4% for women aged 38e39
C
4.7% for women aged 40 and over
In 2010 for stimulated DI that resulted in a live
birth was:
C
20.7% for women aged under 35
C
17.1% for women aged 35e37
C
11.9% for women aged 38e39
C
5.3% for women aged 40 and over
NICE states that there is insufficient evidence
to recommend the use of GIFT in preference to
IVF in couples with unexplained fertility
problems or male factor fertility problems
The chances of success with IVM are similar to
those with conventional IVF, with the risk that
if the eggs are immature it is likely that fewer
eggs will be collected and fewer fertilization

Table 2

uterus and placed 10 mm below the top of the endometrial lining.


HFEA recommends that the best one or two embryos are transferred.
This is the HFEAs limit on numbers for women aged less than 40;
three embryos may be transferred in exceptional circumstances only
in women aged 40 or over. However when donor eggs are used, no

Embryo transfer
The embryo transfer usually takes place on the fifth day after oocyte
retrieval. Embryo transfer is a simple procedure that requires a full
bladder and it is performed without sedation. A small, soft catheter
is loaded with the embryos and inserted through the cervix into the

OBSTETRICS, GYNAECOLOGY AND REPRODUCTIVE MEDICINE 26:7

204

2016 Elsevier Ltd. All rights reserved.

REVIEW

Long and short protocols


LONG PROTOCOL
GnRH agonists

hCG trigger

Gonadotrophins

Day 1
menstruation

Day 21 Day 1 Pituitary


downregulation

Oocyte
retrieval

SHORT PROTOCOL
hCG trigger

GnRH agonists
Gonadotrophins

Day 1
menstruation

Day 35

Oocyte
retrieval

Figure 3

(Figure 6). Younger women are more likely to have a successful


pregnancy. IVF is not usually recommended for women above
the age of 42, because the chances of a successful pregnancy are
thought to be too low.
IVF and intra-cytoplasmic sperm injection (ICSI) success rates
are similar.

more than two can be transferred. This is to avoid the risk of multiple
pregnancy and its sequelae. Any embryos left and of sufficiently
high quality are cryopreserved. There is clear evidence to suggest
that a single embryo transfer in each treatment cycle not only gives a
better cumulative pregnancy rate but also leads to significant
reduction in the worse perinatal outcomes associated with multiple
pregnancies.

Complications of ART
Success

Patients undergoing ovulation induction/stimulation should be


given up to-date information about the long-term health outcomes of these treatments.

The success rate of IVF depends on the age of the woman undergoing treatment, as well as the cause of the infertility

Antagonist protocol

Fixed dose protocol Day 6

Multiple dose protocol Day 8


hCG
trigger
Gonadotrophins

Day 2
menstruation
Day 1
menstruation

Oocyte
retrieval

Figure 4

OBSTETRICS, GYNAECOLOGY AND REPRODUCTIVE MEDICINE 26:7

205

2016 Elsevier Ltd. All rights reserved.

REVIEW

TVUSS guided oocyte retrieval

Ovarian
ligament

Uterus
Fallopian
tube

Ovary
Follicle
Aspiration
needle

Uterine
cervix
Ultrasound
beam

Vagina

Ultrasound
probe

Collection
tube

Suction
pump

Figure 5

incidence of mortality from OHSS is unknown, and possibly


under-reported, deaths from OHSS are rare. Women should be
informed that mild forms of OHSS are common, affecting up to
33% of IVF cycles and that <0.5e2% of IVF cycles are

Multiple pregnancy
This is a major cause of perinatal morbidity and mortality in IVF
pregnancies. This is secondary to increased risk of prematurity,
preterm rupture of membranes and growth restriction and, its
long term sequelae. Multiple pregnancy is also associated with an
increased risk of maternal morbidity (pre-eclampsia, thromboembolic disease, ante partum and postpartum haemorrhage and
caesarean section). Multiple pregnancy rates are currently
around 20% for all IVF pregnancies. A target rate of <10% for
multiple pregnancies for all IVF pregnancies has now been set. It
is envisaged that using better embryo culture and selection
methods, and wider use of single blastocyst transfer could achieve this target. Below is the table comparing the pregnancy rate
between double and single embryo transfer (Table 3).

Prediction of IVF success


Prognostic indicators for IVF

Ovarian hyperstimulation syndrome (OHSS)


OHSS is a systemic disease resulting from vasoactive products
released by the hyperstimulated ovaries. The pathophysiology of
OHSS is characterised by increased capillary permeability, leading to leakage of fluid from the vascular compartment, with third
space fluid accumulation and intravascular dehydration. Severe
manifestations of OHSS include a tendency to develop thrombosis, renal and liver dysfunction and acute respiratory distress
syndrome (ARDS), causing serious morbidity. Although the true

OBSTETRICS, GYNAECOLOGY AND REPRODUCTIVE MEDICINE 26:7

Good prognostic
factors

Bad prognostic
factors

Age < 30
Previous pregnancy
BMI 1930
Unexplained infertility

Poor ovarian response


Previous unsuccessful cycles
Infertility > 6 years
Caffeine > 50 mgs/day
Alcohol > 1 unit/day
Smoking

Figure 6

206

2016 Elsevier Ltd. All rights reserved.

REVIEW

Multiple pregnancy rate


Age

Pregnancy
rate (2013)

eSET
(blastocyst 2013)

Multiple
pregnancy rate
(eSET 2013)

DET (blastocyst 2013)

Multiple pregnancy
rate (DET 2013)

18e34
35e37
38e39
40e42
43e44
45 years
All ages

40%
36%
30%
33%
23%
8%
34%

47%
44%
37%
27%
27%
27%
45%

>2%
>2.5%
>2.5%
>2.5%
>2.5%
>2.5%
>2%

47%
45%
41%
35%
23%
23%
43%

40%
35%
28%
24%
16%
16%
35%

SET, single embryo transfer; DET, double embryo transfer.

Table 3

Clinical picture of OHSS


Clinical symptoms
C
C

C
C
C
C
C
C

Lower abdominal discomfort


Progressive increase in abdominal
circumference
Nausea, vomiting
Diarrhoea
Dyspnoea
Respiratory distress
Rapid weight gain
Oliguria/Anuria

Clinical signs
C
C
C
C
C
C

C
C

Laboratory findings

Weight gain
Hypovolaemia
Low blood pressure
Enlarged ovaries on ultrasound scan
Ascites e clinical/ultrasound
Thromboembolism (specially upper body
veins)
Pleural effusion
Pericardial effusion

C
C
C
C
C
C
C

Haemoconcentration
Leucocytosis
Electrolyte imbalance
Hypoalbuminaemia
High liver enzymes
Low creatinine clearance
Hypercoagulability

Table 4

 Coasting by discontinuing gonadotropins stimulation


while continuing GnRHa and delaying HCG trigger until
oestradiol levels fall below 9000 pmol/litre. This can
reduce the risk of OHSS but leads to a reduced number of
oocytes available and potentially compromises pregnancy
rate.
 Elective cryo-preservation of all embryos by deferring fresh
embryo transfer to another cycle as this can reduce the risk

complicated by moderate or severe OHSS. There are two types of


OHSS; an early type and a late onset type. The symptoms, signs
and laboratory findings of OHSS are summarized in Table 4 and a
classification of the disorder is shown in Table 5.
Risk factors for development of OHSS: young age, lean
physique, polycystic ovary syndrome, high serum oestradiol
>9000 pmol/litre, rapidly increasing oestradiol levels (>75%
from previous day), use of HCG as a trigger, number of oocytes
retrieved (>20), and multiple pregnancies.
The syndrome occurring within 10 days of use of HCG is
called early type OHSS and is usually milder in severity and selflimited within 14 days if pregnancy does not occur.
The late type OHSS usually presents >10 days after use of
HCG, is commonly associated with multiple pregnancy and tends
to have a more severe and protracted course but eventually resolves in early pregnancy after 4e6 weeks.

Classification of OHSS

Prevention of OHSS: the following preventive strategies have


been shown to be partially effective in preventing or ameliorating
OHSS:
 Cycle cancellation by withholding the ovulatory trigger of
HCG, the risk of OHSS is almost negligible, however this is
only practical where GnRH agonists have been used and is
considered a failed cycle.

OBSTETRICS, GYNAECOLOGY AND REPRODUCTIVE MEDICINE 26:7

Ovarian size

Signs and symptoms

Mild

<8 cm

Moderate

8e12 cm

Severe

>12 cm

Mild pain/discomfort
Abdominal swelling
Abdominal pain/discomfort,
Nausea/vomiting
Ascites (USS)
Pain, dyspnoea/tachycardia
Oedema
Oliguria
Ascites (Clinical/USS)
Pleural effusion

Table 5

207

2016 Elsevier Ltd. All rights reserved.

REVIEW

of late OHSS but does not prevent onset of early type of


OHSS.
Prophylactic albumin administration around oocytes
retrieval in high risk women. This acts by binding vasoactive substances and preventing fluid shift into the third
space by increasing intravascular oncotic pressure. The
optimal dose and timing of albumin administration are not
clear and there are concerns about febrile and allergic reactions, and potential risk of virus/prion transmission.
An alternative to albumin is hydroxyl ethyl starch, which
is a plasma substitute and may be safer, cheaper and
effective.
Cabergoline is a dopamine agonist and also inactivates
VEGF receptor 2. Cabergoline in a dose of 0.5-mg/day daily
from the day of HCG administration for 8e10 days has
been shown to reduce risk of moderate OHSS by 60% but
not of severe OHSS.
Use of GnRH antagonists for pituitary suppression have
been associated with lower risk of OHSS by 40% and cycle
cancellation compared with GnRH agonist protocols. It is
also possible to reduce the risk of OHSS further by
substituting HCG with GnRH agonist as a trigger.
Other methods used in patients with polycystic ovary
syndrome include: Metformin has been shown to be
effective in reducing risk of OHSS and cycle cancellation.
Ovarian drilling prior to start of ovarian stimulation reduces risk of severe OHSS especially in young patients with
PCOS.

In severe to critical OHSS consider MDT input with ITU care.


If Hct >45% consider intensive rehydration (1 litre/hour)/Paracentesis/Drainage of hydrothorax/Surgery for adnexal torsion
depending on clinical condition.

Investigations for OHSS


 Full blood count (haematocrit/WCC/platelets)
 U & Es, Albumin and LFTs
 Ultrasound of abdomen/pelvis
 If dyspnoea/tachycardia e CXR (and consider O2 saturation monitor or blood gases) and ECG
 If severe e clotting studies

The HFEA is the regulating body in the UK for treatment and


research using eggs, sperm and human embryos. They set standards and give guidance to all the fertility clinics in the UK. They
conduct regular inspection of all the UK fertility clinics and
research centres. This helps to ensure every licensed clinic or
centre is adhering to standard safety and ethical rules set out by
the UK government.

Management of OHSS
Analgesia
 Paracetamol or co-codamol (avoid NSAIDs)
 Parenteral opiates may be required but the patient must be
reviewed to exclude additional pathology (ovarian cyst
rupture, haemorrhage or torsion, etc).
Antiemetics
 Metoclopramide or prochlorperazine
Thromboembolic prophylaxis
 TED stockings
 Prophylactic subcutaneous Fragmin should be considered
for all admitted patients e 2500 IU subcutaneously daily
or, if more severe, may need 5000 IU.
 Formal anticoagulation is reserved for those cases with
clinical evidence of thromboembolism.

Pre-implantation genetic diagnosis (PGD)

Cancer
There is no direct association between infertility management
regimens used and the development of invasive cancer. The
ovulation induction/stimulation agents should be used to the
lowest effective dose and duration. Women should be counselled
regarding the uncertainty of the current evidence and that there
may be increased risk of borderline tumours.
Fetal and neonatal complications
There is no evidence of increased risk of childhood illnesses or in
neuro-developmental or psychomotor developmental problems
in IVF babies. There is inconclusive evidence that IVF/ICSI may
increase the risk of particularly imprinting disorders. Certain
genetic and developmental defects have been reported in a very
small number of children born using ICSI treatment. It is still not
clear whether these risks are associated with the procedure or to
inherent sperm abnormalities. Undescended testis and urogenital
anomalies were reported in boys conceived by ICSI. A recent
study (2013) of 106,013 children born after assisted conception
found no increased risk of cancer in ART children. Long-term
safety of new technologies that are being introduced should be
established by setting up a national database.

The Human Fertilization and Embryology Authority (HFEA)

Since its inception in 1990s, PGD following IVF has allowed


embryo genetic diagnosis before implantation thereby eliminating the risk of termination at a later stage of pregnancy. This
has been valuable in couples with structural chromosomal and
mitochondrial disorders and single gene defects including cystic
fibrosis, the most common autosomal recessive disorder. Biopsy
of a cell or two at 8-cell stage of the embryo does not adversely
affect the preimplantation development allowing a 8e12 hour
genetic analysis before transfer of a selected embryo. Clinical
outcome data are collected by the Preimplantation genetics international society and European Society for Human Reproduction and Embryology (ESHRE) PGD consortium.

Oocyte donation

Fluid balance
 Oral fluids should be encouraged for all patients
 Intravenous fluids reserved for patients who are admitted
(2e3 litres/day)
 If urine output <700 ml/24 hours e catheterise and
monitor hourly urine volumes.

OBSTETRICS, GYNAECOLOGY AND REPRODUCTIVE MEDICINE 26:7

Indications for oocyte donation include premature ovarian failure, gonadal dysgenesis, poor egg quality and diminished
ovarian reserve. This should be discouraged if there are underlying medical problems. The risk of genetic abnormalities in the
offspring of women undergoing oocyte donation relates to the

208

2016 Elsevier Ltd. All rights reserved.

REVIEW

age of the donor. Appropriate counselling of all couples considering these procedures should be arranged prior to embarking on
the process.

RCOG. The management of ovarian hyperstimulation syndrome.


Green top guideline No.5. Royal College of Obstetricians and
Gynaecologists, 2006.
World Health Organization. WHO laboratory manual for the examination of human semen and sperm-cervical mucus interaction. 5th
edn. World Health Organisation, 2010.

Surrogacy
This is defined as Another woman carries and gives birth to a
baby for the couple who want to have a child. Some of the indications for surrogacy include recurrent miscarriages, uterine
malformations and repeated IVF implantation failures. The HFEA
does not regulate surrogacy. In the UK, the surrogate will be the
legal mother of the child unless or until parenthood is transferred
to the intended mother through a parental order. It is advised that
the couple should seek legal advice before proceeding with this
option. Surrogacy UK is a non-profit organization offering support and information to anyone with an interest in surrogacy
within the UK.
A

Practice points
C

FURTHER READING
Fertility: assessment and treatment of people with fertility problems
NICE clinical Guideline (CG156). 2013, http://www.nice.org.uk/
guidance.
Human Fertilisation and Embryolgy authority. http://www.hfea.gov.uk/.
Improving outcomes for fertility patient: multiple births. HFEA, 2015.
Mahmoud Mohammed K, Punukollu Durgadevi, Mahmood Tahir.
In vitro. Fertilization. Obstet Gynaecol Reprod Med 2013; 23:
238e46, http://www.obstetrics-gynaecology-journal.com/.
van Loendersloot Laura, Repping S, Bossuyt PMM, van der Veen F,
van Wely M. Prediction models in in vitro fertilization; where are
we? A mini review. J Adv Res May 2014; 5: 295e301.

OBSTETRICS, GYNAECOLOGY AND REPRODUCTIVE MEDICINE 26:7

C
C

209

It is estimated that infertility affects 1 in 7 heterosexual couples in


the UK. In the UK 224,196 babies were born after IVF treatment
between 1991 and 2011
Couples attending for their initial consultation before embarking
on IVF treatment should be given lifestyle advice, offered general
screening and specific infertility investigation work up
The treatment cycle comprises of seven main steps including pituitary down regulation, ovarian stimulation, monitoring of
response to ovarian stimulation, ovulation trigger, oocyte
collection, luteal support and embryo transfer
The complications of ART include OHSS and multiple pregnancy
The HFEA is the UKs independent regulator of treatment using
eggs and sperm, and of treatment and research involving human
embryos
The success rate of IVF depends on the age of the woman undergoing treatment, as well as the cause of the infertility. Younger
women are more likely to have a successful pregnancy

2016 Elsevier Ltd. All rights reserved.

ETHICS/EDUCATION

Magnesium sulphate for


neuroprotection

compared to gestational age matched controls (7% vs 36%, OR


0.14; 95% CI 0.05e0.51). This was certainly plausible. MgSO4
had been used successfully as a maternal neuroprotective agent
in pre-eclampsia and eclampsia. Magnesium through its action
on N-methyl-D-aspartate (NMDA) receptor, prevents hypoxia
mediated calcium influx which leads to cell death. It decreases
free radical production during hypoxic ischaemic reperfusion and
is a potent cerebral vasodilator. Magnesium, when given to the
mother, is found in placental circulation within an hour and
amniotic fluid within 3 hours, suggesting the fetus metabolizes
and excretes it similarly and may benefit from similar neuroprotection. Animal studies confirmed this and human trials then
started.
The three trials that looked at neuroprotective intent were
ACTOMgSO4 from Australia, PREMag from France and BEAMs
from USA. All three compared MgSO4 to placebo and were given
to women at risk of preterm delivery within 24 hours. All three
trials had outcomes of CP and death which were lower in the
MgSO4 groups but none were statistically significant. The Magpie
trial was primarily designed to determine maternal neurological
effects of magnesium but also reported some neonatal outcomes.
Because of sheer volume, it provided enough data to suggest a
reduced risk of CP and death in those who were allocated MgSO4.
All these trials are summarized in Table 1. The MAGNet trial,
which had two arms, tocolysis and neuroprotection, had to be
stopped midway because there were reports of an excess of
15.2% neonatal mortality. These were all in the tocolytic arm
where the total median exposure of MgSO4 was 48 g compared to
the other trials (PREMAG 4 g, ACTOMgSO4 11 g, BEAMs 31 g)
where there was no significant difference in mortality.
A Cochrane meta analysis of these five trials subsequently
gave us the evidence we needed. It demonstrated that antenatal
MgSO4 given to women at risk of delivery before 37 weeks
significantly reduced the risk of CP (RR 0.68; 95% CI 0.54e0.87)
and gross motor dysfunction (RR 0.61; 95% CI 0.44e0.85) with
no increase in mortality. Based on this the NNT to benefit one
baby before 37 weeks was 63 (95% CI 43e155). When Conde
Agudelo and Romero reviewed the data for babies under 34
weeks, the calculated NNT was 52.

Shilpa Nambiar

Abstract
Magnesium sulphate has been established as an effective maternal
neuroprotective agent in pre-eclampsia and eclampsia and evidence
has shown that antenatal administration to mothers at risk of preterm
delivery reduces the risk of perinatal cerebral injury and subsequent
cerebral palsy and gross motor dysfunction. This article examines
the available evidence, its interpretation and its translation into clinical
practice.

Keywords cerebral palsy; magnesium; neuroprotection; preterm

Introduction
Cerebral palsy (CP) is a symptom complex of non-progressive
motor impairment syndromes secondary to brain injury or
anomalies arising in early development. The prevalence is estimated at 2 per 1000 live births, with a quarter of these attributed
to prematurity under 34 weeks. Improved neonatal facilities have
improved survival rates but has increased the incidence of cerebral palsy in survivors.
The aetiology for CP is multifactorial but very low birth
weights and extreme prematurity account or up to a third of
cases. The premature brain is exquisitely sensitive to white
matter injury because of vascular instability of the germinal
matrix leading to haemorrhage, hypoxia and subsequent cytokine and free radical activation. The most frequently observed
abnormalities associated with CP are peri-intraventricular haemorrhages (PIVH) and periventricular leukomalacia (PVL). Both
these complications affect the pyramidal tracts of lower extremities and lead to spastic CP of the legs.
PIVH originates in the vascular bed of the germinal matrix, an
area or fetal brain that completely disappears as the fetus matures. Hence, the incidence of CP decreases significantly with
increasing gestation: 14.6% at 22e27 weeks, 6.2% at 28e31
weeks, 0.7% at 32e36 weeks and 0.1% at term.

Why the hesitation?


Despite this apparently strong statistical support for its use and
guidelines developed in several countries to incorporate it into
clinical practice, uptake has been slow and cautious.
Part of the reason may be the adverse outcomes reported by
Mittendorf. More recently the FDA issued a warning against
prolonged administration of MgSO4 as it was found to be associated with skeletal abnormalities. But again, these were cases
where exposure was in excess of 3700 g.
Several authors have also questioned the validity of the results from the meta analysis. Their argument is that when Trial
Sequential Analysis is used to assess data from randomized
controlled trials, the apparently beneficial effect may be due to an
increase in random error. The solution is to either conduct a
bigger trial or to do another meta analysis using individual data
from the original RCTs. Crowthers group has undertaken just
this with AMICABLE and we eagerly await the results. Meanwhile, investigators from Denmark, Sweden, Norway and Iceland

Evidence for magnesium for neuroprotection


The concept of magnesium sulphate (MgSO4) for neuroprotection
was sparked in 1995 by Nelson and Grether, two epidemiologists
who did a case control analysis of more than 150,000 children
born weighing less than 1500 g and followed them up to age 3.
They found that if mothers had been exposed to MgSO4 antenatally, their children were significantly less likely to have CP

Shilpa Nambiar MD MRCOG MRCPI is Associate Professor and Clinical


Lead Obstetrics and Gynaecology, Consultant Obstetrician and
Gynaecologist at Perdana University, Royal College of Surgeons
Ireland School of Medicine, Prince Court Medical Centre, Kuala
Lumpur, Malaysia. Conicts of interest: none.

OBSTETRICS, GYNAECOLOGY AND REPRODUCTIVE MEDICINE 26:7

222

2016 Elsevier Ltd. All rights reserved.

ETHICS/EDUCATION

The randomized controlled trials in the meta analysis


TRIAL, author
(n: women), year

Eligibility (n: fetuses)

MgSO4 dosage used

Cerebral palsy risk

CP/death risk

ACTOMgSO4,
Crowther et al. (1062), 2003
PREMAG, Marret
et al. (564), 2007
BEAMs, Rouse
et al. (2241), 2008
MAGNet, Mittendorf
(149), 2002

<30 weeks (1255)

4 g bolus, 1 g/hour

RR 0.51; 95% CI 0.29e0.91

RR 0.82; 95% CI 0.66e1.02

<33 weeks (688)

4 g bolus

RR 0.69; 95% CI 0.49e1.19

RR 0.80; 95% CI 0.58e1.10

24e32 weeks (2444)

RR 0.55; 95% CI 0.32e0.95

RR 0.90; 95% CI 0.73e1.10

MAGPIE, Collaborative
Group (1544), 2002

<37 weeks (1593)

6 g bolus, 2 g/hour
with retreatment
4 g bolus, 2e3 g/hour
(tocolysis) 4 g bolus
(neuroprotection)
4 g bolus, 1 g/hour or
5 g IM every 4 hours

25e33 weeks (165)

RR 4.85; 95% CI 0.60e38.90

RR 0.83; 95% CI 0.66e1.03

Table 1

have started recruitment for what they deem to be the final to


answer these questions.
There is also uncertainty about the gestation under which
MgSO4 should be offered. The Australian recommendations use a
cut off of 30 weeks and the Canadian society advocates 32 weeks.
The ideal cut off would benefit the largest number of women
while avoiding unnecessary administration to those least at risk.
Institutions should make their own recommendations according
to resource limitations.
There is no consensus about dosing, regiments and repeat
administration. There is evidence of harm with high doses used
for tocolysis and similar benefits when the lower dosage used for
eclampsia prevention. It would seem prudent to adopt the regime
and monitoring which most obstetricians are already familiar
with to gain the same risk reduction for CP. Based on this FIGO
recommends giving 4e6 g as a bolus followed by 1 g/hour for a
maximum of 24 hours with no repeats.

OBSTETRICS, GYNAECOLOGY AND REPRODUCTIVE MEDICINE 26:7

Conclusion
Despite all the unanswered questions, there is substantial data to
support the use of magnesium sulphate as a tool to reduce the
risk of morbidity from preterm births. It is hoped that there will
be more acceptance once results from ongoing studies are
available.
A
FURTHER READING
Doyle LW, Crowther CA, Middleton P, Marret S. Antenatal magnesium
sulfate and neurological outcome in preterm infants: a systematic
review. Obstet Gynecol 2009; 113: 1327e33.
Magnesium sulphate for fetal neuroprotection: guidelines consensus
group. J Obstet Gynaecol Can 2011; 33: 516e29.
Royal College of Obstetricians and Gynaecologists Scientic Impact
Paper No. 29: Magnesium Sulphate to Prevent Cerebral Palsy
Following Preterm Birth, Aug 2011.

223

2016 Elsevier Ltd. All rights reserved.

CASE-BASED LEARNING

The subfertile couple

The management of a subfertile couple starts in primary care


with a detailed medical, gynaecological, social and family history followed by a general and pelvic examination. Other pathologies are occasionally present and it should always be borne
in mind that subfertility might present the opportunity to detect
and treat serious or life-threatening co-morbidity. For example,
azoospermia or severe oligozoospermia may be the presentation
of a testicular or adrenal androgen-secreting tumour; or side
effects of medications (Table 1). Similarly, gonadal or adrenal
tumours may rarely present with anovulatory subfertility,
particularly if associated with a high blood testosterone levels
and rapidly progressive features of hyperandrogenisation or
virilisation in younger women. Pelvic endometriosis may present with chronic pain and subfertility from severe tubo-ovarian
disease, the surgical treatment of which may predispose to
premature menopause. Tuberculosis is becoming more common
in the UK with the increase in human migration. This disease
may present for the first time with chronic tubal disease in a
subfertile couple.
General lifestyle advice should be offered on the effects of
obesity, cigarette smoking, occupational hazards, alcohol or
recreational drug misuse on fertility. Baseline tests include
semen analysis, cervical smear test, screening for chlamydia,
tests of ovulation and tubal patency. Semen analysis should
ideally be performed in dedicated andrology laboratories with
semen parameters measured against the World Health Organisation (WHO) reference values (Table 2). Semen analysis should
be repeated approximately three months after an abnormal
result. If severely abnormal, further tests are mandatory as
shown in case 1. Occasionally a very low ejaculate volume may
be associated with azoospermia, or severe oligozoospermia, in
which case the diagnosis of retrograde ejaculation should be
suspected, leading to the analysis of a post-ejaculate urine
sample.
The majority of regularly menstruating women are ovulating,
therefore a single assay of mid-luteal phase progesterone is an
adequate test of ovulation in the UK. However, in women with
irregular menstrual cycles, it can be difficult or impossible to
time a mid-luteal progesterone assay. For this group, anovulation
should be suspected therefore, assays of follicle stimulating
hormone (FSH) and luteinising hormone (LH) should be obtained in the basal (proliferative) phase of the menstrual cycle.
For women older than 37 years, it may be prudent to assay FSH
as a baseline test of ovarian reserve. Unless indicated by suspicions of co-morbidities from the history, assays of prolactin, or
thyroid function tests are not routine part of basic subfertility
investigations (Table 3).
The default test for tubal patency is hysterosalpingogram
(HSG). Other credible alternatives, where the skills are available,
are selective salpingogram and tubal catheterisation, and
hysterosalpingo-contrast-sonograpgy (HyCoSy). However, diagnostic laparoscopy and dye test is the preferred choice, if there
are clinical suspicions of pelvic inflammatory disease, endometriosis, previous pelvic surgery or ectopic pregnancy.
Subfertility and its management can present the couple with
significant stress and anxiety, which may adversely affect their
physical relationships. They should therefore be offered counselling services at any stage of their management, particularly if
any assisted conception treatment is being considered. There is a

Mamoun Bereir
Carol Coughlan

Abstract
Subfertile couples by denition require medical help to get pregnant
after trying unsuccessfully for a variable period of time. Although the
term subfertility is also occasionally used in the context of women
who can conceive, but suffer recurrent miscarriages, this broad aspect
is outside the scope of this chapter. Subfertility can seriously affect
mental and social well-being, although not generally viewed as a disease that signicantly causes physical ill health. On the contrary, subfertility may in fact be the early manifestation of serious co-existing
disease. Advances in assisted reproductive technology in the last 25
years have simplied and diversied treatment options, thereby
rendering the terms infertility and sterility unfashionable. As a result,
there is now an unfortunate trend for couples to be subjected to supercial medical history and only perfunctory physical examinations, occasionally missing the presentations and implications of associated
co-morbidities, with devastating implications. There are now many
guidelines on the initial investigations and subsequent management
of the subfertile couple. It is therefore now relatively easy to assist a
couple to achieve a pregnancy. However, dealing with the devastating
news of causative factors like azoospermia, genetic disease, congenital anomaly or premature ovarian failure can be very difcult. To
discuss these sympathetically, professionals require interpersonal
skills in breaking bad news. Finally, a multidisciplinary team approach
should be adopted to cater for the long-term health consequences,
whenever co-morbidities are detected.

Keywords anovulation; azoospermia; fallopian tube diseases;


hydrosalpinx; infertility; IVF; oligospermia; PCOS

Subfertility affects approximately one in seven couples. One of


the measures of fecundity (ability to reproduce) is fecundability
(the monthly probability of pregnancy); which is only about
15%. Theoretically therefore, in women under the age of 36
years, the cumulative probability of pregnancy is 60% at 6
months of trying, about 85% at the end of one year and 95% by
the end of the second year. Other factors can affect these rates
however; including age of female partner over 36 years and
presence of co-morbidity. The distribution of causative factors
varies slightly from one region to another. In the UK about 25%
of cases, have no identifiable causes (unexplained), whereas
male factors can account for up to 30%. The breakdown of female factors is shown in Figure 1. In about 20% of cases, both
male and female factors may co-exist.

Mamoun Bereir MD MRCPI is a Clinical Fellow in Reproductive


Medicine at Rotunda IVF, Rotunda Hospital, Dublin, Ireland. Conicts
of interest: none declared.
Carol Coughlan FRCOG FRCPI MD is a Subspecialist in Reproductive
Medicine and Surgery at Rotunda IVF, Rotunda Hospital, Dublin,
Ireland. Conicts of interest: none declared.

OBSTETRICS, GYNAECOLOGY AND REPRODUCTIVE MEDICINE 26:7

210

2016 Published by Elsevier Ltd.

CASE-BASED LEARNING

Case 1
25%

Miss NW, aged 34 years and her partner Mr JG, aged 41 years,
were referred to the local teaching Hospital from the district
Hospital on account of secondary subfertility due to tubal disease
of 12 months duration. They had been together for over two
years. Coital frequency was at best twice a week because of the
association with deep dyspareunia. Miss NW had a child aged 10
years from a previous relationship who lived with them. She had
separated from her previous partner because of infidelity
resulting in Chlamydia pelvic inflammatory disease, which was
eventually treated after some delay in diagnosis. She was on folic
acid, immune to Rubella, up to date with her cervical smear tests
and had been advised to cut down on alcohol and her 25 cigarettes per day habit. Hysterosalpingogram (HSG) performed at
the referring hospital showed bilateral large hydrosalpinges, with
no peritoneal spillage on either side. All other fertility tests were
normal. The couple had been told by the referring doctor that she
had two options of definitive treatment; either tubal surgery or
in-vitro fertilisation (IVF).
 What initial advice would you give to this couple?
 What is your opinion about the choice of tubal patency test
in this case?
 What are the treatment options?
Pre-conception advice about diet, weight reduction, sensible
use of alcohol, cigarette cessation and prophylactic folic acid is
good practice during the first visit. Many couples also need to be
advised on a coital frequency of about 3 times a week, not timed
to ovulation unless medically indicated.
Most gynaecologists will disagree with the choice of HSG as
the test of tubal patency for this patient. Whenever there is a
reliable history of sexually transmitted disease (STD), or there
was co-existing pelvic pain or dyspareunia, diagnostic laparoscopy and dye test is certainly a better choice as it allows
assessment of the extent of tubal damage and inspection of the
peritoneum for adhesions or endometriosis. Diagnostic laparoscopy allows staging of the degree of tubal disease in order to
formulate a prognosis. For stage 1 (thin-walled with little or no
fibrosis) and 2 (thick-walled with good mucosa), laparoscopic
adhesiolyses and cuff salpingostomies performed at the same
time could offer a clinical pregnancy rate (CPR) of up to 50%
over three years; whereas, for stages 3 (thick-walled with marked
mucosal damage, or a thick fibrous endosalpingeal adhesion)
and 4 (tubo-ovarian mass or fibrosis, or an adherent hydrosalpinx with incarcerated ovary and/or isthmic damage), the
rates over the same period were less than 12%. Furthermore,
ectopic pregnancy rates after tubal surgery for stages 3 and 4
were as high as 25%.
The other treatment option was IVF. However, there is evidence that untreated hydrosalpinges or unilateral tubal disease,
which communicates with the uterine cavity, could significantly
reduce clinical pregnancy rates. Hydrosalpinges adversely affect
endometrial receptivity, thereby decreasing the implantation rate
and increasing the risk of early miscarriages. There is also evidence showing that laparoscopic salpingectomy or salpingectomies could more than double the live birth rate following IVF
to about 30% per cycle in good centres.
She underwent diagnostic laparoscopy and staging of tubal
disease, with additional consent to proceed to either adhesiolysis

30%

10%
20%

Male factors
Ovulatory disorders
Unexplained

15%

Tubal disease
Uterine/peritoneal

Figure 1 Causes of subfertility.

very wide spectrum of treatment options for subfertility,


depending on mode of presentation, the duration of onset, age of
the female partner and type of co-morbidity. It is not possible to
deal with all treatment options of subfertility in this chapter;
however the following cases are presented to illustrate the clinical presentations of a spectrum of subfertile couples.

Classification of azoospermia by causation or


co-morbidity
Pre-testicular (hypogonadotropic hypogonadism) azoospermia
C
Use of anabolic steroids (blood testosterone may be high if
Sustanon is used)
C
Extra-gonadal androgen producing tumours
C
Pituitary tumours e.g. prolactinoma, craniopharyngioma, cancer
C
Kallmans syndrome
C
Haemochromatosis (iron overload)
Testicular azoospermia
C
Trauma, torsion,
C
Undescended testes (Cryptorchidism)
C
Chromosomal (aneuploidies)
 Klinefelters syndrome
C
Genetic disorders/mutations
 5 Alpha reductase deficiency
 Androgen insensitivity syndromes
 Sertoli only syndrome
C
Infection involving the testes (mumps orchitis)
C
Androgen producing testicular tumours
C
Iatrogenic
 Surgery, radiation or chemotherapy
Post testicular azoospermia
C
Post infective: chlamydia, gonorrhoea, tuberculosis
C
Iatrogenic: post hernia or hydrocoele surgery, vasectomy
C
Genetic mutations
 Cystic fibrosis
 Congenital bilateral absence of the vas deferens
C
Ejaculatory failure or retrograde ejaculation
Table 1

OBSTETRICS, GYNAECOLOGY AND REPRODUCTIVE MEDICINE 26:7

211

2016 Published by Elsevier Ltd.

CASE-BASED LEARNING

to discuss the prognoses further. The clinical indications for IVF


became much clearer and they eventually returned for laparoscopic ovariolyses and bilateral salpingectomy. Histology
confirmed chronic endosalpingitis with no evidence of malignancy. She underwent IVF, which unfortunately resulted in a
biochemical pregnancy. However, she subsequently had a successful frozen embryo transfer cycle resulting in a live born baby
girl.

WHO 2010 reference values for semen parameters


Semen parameter

Normal ranges

Semen volume (mls)


Sperm concentration (million per mL or more)
Total sperm number (million per ejaculate)
Progressive motility (PR, %)
Total motility (PR NP, %)
Vitality (live spermatozoa, %) S
Sperm morphology (normal forms %)

1.5
15
39
32
40
58
4

(1.4e1.7)
(12e16)
(33e46)
(31e34)
(38e42)
(55e63)
(3.0e4.0)

Case 2
A Polish couple were referred to a tertiary centre with a history of
inability to conceive of 24 months duration. Both were 28 years
old and in good health. Neither smoked cigarettes, but both
drank roughly 10 units of alcohol per week. Mrs AT had a regular
menstrual cycle, an up-to-date normal cervical smear test, body
mass index (BMI) of 27, was rubella immune and had screened
negative for Chlamydia infection. Physical examination was unremarkable. Her mid-luteal progesterone assay was 63 nmol/L.
Mr PT was in good health, worked as a doorman in a nightclub,
and at a building site during the day. He had never fathered a
child. He had a right inguinal herniorrhaphy performed in Poland
5 years previously. He denied any significant drug history. Semen
analysis showed azoospermia on two occasions. Examination
revealed a fit, muscular young man with a normal phallus.
Testicular volumes were about 15 mL bilaterally. The epididymis
and vas were easily palpable bilaterally. No nodules or irregularity were detectable on either side.
Blood tests on Mr PT were reported as below.

Table 2

and cuff salpingostomies for stages 1e2. If however stage 3e4


disease was found, the couple wanted more time to consider the
medical, social and financial implications of IVF, before returning for salpingectomies. The couple had access to counselling
services. She eventually underwent staging laparoscopy, during
which bilateral large tubo-ovarian masses were recorded. No
further action was taken at the time according to the wishes of
the couple. They returned to follow-up clinic three months later

Ovulatory disorders classified by causation and


co-morbidity
WHO Group I: (Low FSH and oestrogen levels)
C
Hypothalamic/pituitary hypogonadism (hypogonadotropic)
 Weight loss, intense exercise, chronic illness
 Genetic e.g. Kallmanns syndrome
 Idiopathic, psychological
 Head injury
 Tumours e.g. craniopharyngiomas, gliomas
 Iatrogenic e.g. surgery, cranial irradiation
WHO Group II: (Normal FSH and oestrogen levels)
C
Hypothalamic pituitary dysfunction (Normogonadotropic)
 Polycystic ovary syndrome
 Weight related anovulation
WHO Group III: (High FSH and low oestrogen levels)
C
Premature ovarian failure (hypergonadotropic)
 Chromosomal e.g. Turners syndrome and Turner mosaic
 Genetic mutations e.g. micro deletions X chromosome, FSH
receptor gene mutation, Fragile X syndrome
 Infective e.g. mumps-oophoritis
 Iatrogenic causes e.g. surgery, radiation or chemotherapy
Other endocrinopathies
Excessive prolactin e.g. prolactinoma
Excessive growth hormone e.g. acromegaly
Excessive androgens: Late onset congenital adrenal hypertrophy
(LoCAH),
: Pituitary, adrenal or gonadal tumours,
: Disorders of adrenocortical function
C
Cushings syndrome
C
Addisons disease
C
Thyroid disorders
C
Hypothyroidism/Hyperthyroidism

Testosterone
FSH
LH
Karyotype
Cystic fibrosis test

 What conditions in Mr PT could produce the unusual


clinical and endocrine profiles shown above?
 What urgent steps must be taken?
Mr PTs hormonal profile, which showed elevated blood
testosterone level and subnormal pituitary gonadotropins is
suggestive of one of three theoretical possibilities.
(i) Androgen-secreting tumour
(ii) Use or abuse of anabolic steroids or androgens
(iii) Androgen insensitivity syndrome (which is associated
with ambiguous genitalia in the partial variant)
Therefore, in a situation like this the patient must be contacted quickly to review all clinical features, or if indicated
referred for urology opinion, in order to exclude a malignancy.
The couple were immediately recalled to clinic, during which
Mr PT admitted to having self-medicated with testosterone ester
marketed as Sustanon (Organon), the anabolic steroids Nandrolone (Deca-Durabolin e Organon) and Trembolone acetate
(Parabolan) at various times in the preceding 2 years. These
drugs were not medical prescriptions. He claimed to have bought
them over the Internet to boost his bodybuilding regime, and
enhance the ability to perform his job as a nightclub doorman.
The couple claimed ignorance of possible serious side effects. On

Table 3

OBSTETRICS, GYNAECOLOGY AND REPRODUCTIVE MEDICINE 26:7

76 nmol/L (Normal range 8.4e35.7)


<0.3 I.U./L (Normal range 2.5e10.2)
<0.1 I.U./L (Normal range 1.9e12.5)
46 XY
Negative for common mutations

212

2016 Published by Elsevier Ltd.

CASE-BASED LEARNING

deodorants from the age of 9 years. Furthermore, she had oligomenorrhoea, having only ever managed 2e6 menstrual cycles
per year. In the last 6 years, she had to regularly shave her face,
chest and limbs. Her other secondary sex characteristics were
normal. From the preceding year, her hair had started to recede
at the front. There was no significant family, social or drug history. She was clinically obese around her trunk (BMI 37), but
normotensive. Baseline tests carried out at their district hospital
were as shown below.

the contrary, as they had equated increased libido with improved


fertility, Mr PT was encouraged by his wife to resume dosing
after the first sperm count returned very low counts.
 What are the effects of anabolic steroids and androgens on
fertility?
 What other side effects should the couples attention be
drawn to?
By suppressing pituitary gonadotropins via the negative
feedback loop, injected androgens and anabolic steroids can
inhibit spermatogenesis. This is similar to the effects that an
androgen-secreting tumour will have on the hypothalamicpituitary-gonadal axis. Anabolic steroids, which are not converted to testosterone, would give a hypogonadotropic hypogonadic picture, whereas, testosterone esters like Sustanon, as
demonstrated by this case, could give a confusing endocrine
profile with very high serum levels of testosterone; which masks
the true hypogonadism. Prolonged use of anabolic steroids can
also cause abnormal liver enzymes, cholestasis or liver tumours.
An apparently obvious diagnosis should not however obviate the
need to exclude other possible causes of male subfertility (see
Table 1), or a referral to the urologist if indicated.
The couple were informed that his sperm count might fully
recover, but this may take longer than 6 months after stopping
self-medication. Mr PT was advised to stop using the offending
drugs, and return for a repeat sperm test at 6 and 12 months. On
each occasion, his hormonal profiles would also be tested to
monitor progress. Mrs AT was advised to continue to take folic
acid as her fecundability (monthly probability of pregnancy)
could increase with time, if her partner complied with medical
advice. The couple were offered counselling services.
 What is the prognosis?
 What options are available if spermatogenesis failed to
recover?
On few occasions, the testes may fail to recover with hypogonadotrophic hypogonadism persisting after a period of at least
3 months. Such men need urological evaluation to exclude
androgen secreting tumours. Once these are excluded, subfertility may be treated with an antioestrogen like Tamoxifen or
Clomiphene. Antioestrogens interfere with the normal negative
feedback of sex steroids at hypothalamic and pituitary levels,
thus stimulating endogenous hypothalamic gonadotropin
releasing hormones (GnRH) and pituitary gonadotropins (FSH
and LH). Treatment may also be by pulsatile GnRH or a combination of human menopausal and chorionic gonadotropins
(HMG and HCG). If recovery is only partial, or loss of libido
impairs chances of spontaneous conception, intra-uterine
insemination (IUI) or in-vitro fertilisation must be considered.
Intracytoplasmic sperm injection (ICSI) will be the choice of
treatment if total sperm concentration is below 5 million per ml.
The couple returned to clinic as planned. Mr PTs hormonal
profile had normalised by 6 months and spermatogenesis fully
recovered by the end of 12 months.

Semen analysis
HSG
Basal FSH
Basal LH
Testosterone (total)
SHBG

 What is the Free Androgen Index (FAI), and of what


diagnostic value?
Testosterone is mainly carried in the blood bound strongly to
Sex Hormone Binding Globulin (SHBG). A small fraction is carried either in the free form, loosely bound to albumin or CortisolBinding Globulin (CBG). True androgen status can be derived by
measuring the free fraction or by calculating the FAI from total
testosterone and the SHBG (FAI Total Testosterone  100/
SHBG). The FAI for this patient was therefore 5.3. Early proliferative phase FAI greater than 5 is a reliable indicator of
biochemical hyperandrogenaemia, which has been shown to
correlate with clinical hyperandrogenism.
 What differential diagnoses should be considered in this
woman?
 What further investigations are relevant before considering
treatment of subfertility?
The diagnosis of PCOS requires at least 2 criteria from the
following three:
(i) oligomenorrhoea or anovulation,
(ii) clinical or biochemical features of excessive androgens
(iii) polycystic ovaries (PCO) on transvaginal high resolution
ultrasound.
In addition, three differential diagnoses must be excluded; a
step of particular importance for this young obese woman with
early onset disease and FAI greater than 5.0:
(i) late onset congenital adrenal hyperplasia (LoCAH),
(ii) Cushings syndrome
(iii) androgen secreting tumours.
The commonest enzymatic defect in CAH is 21 Hydroxylase
deficiency. Therefore, serum 17-hydroxyprogesterone (17-OHP)
is usually elevated by over 50 times in the early onset CAH, but
much less so in the milder LoCAH. If the diagnosis remains unclear, a referral should be made to a reproductive endocrinologist. Overnight dexamethasone suppression test is the most
reliable screening method for Cushings syndrome. If overnight
suppression of serum cortisol is not achieved, referral to an
endocrinologist is also warranted. Androgen secreting tumours
usually present with rapidly evolving clinical features and much
higher levels of serum androgens (Testosterone, Androstenedione and Dehydro-epiandrosterone sulphate e DHEA-S).

Case 3
A young couple was referred to the local tertiary centre for oligomenorrhoea and anovulatory subfertility probably due to
polycystic ovary syndrome. Mrs AT was 23 years old and
developed armpit hair by the age 8 years and had to use

OBSTETRICS, GYNAECOLOGY AND REPRODUCTIVE MEDICINE 26:7

Normal parameters
Normal cavity and bilaterally patent tubes
3.4 I.U./L (Normal range 2.5e10.2)
8.6 I.U./L (Normal range 1.9e12.5)
4.5 nmol/L (Normal range 0e2.5)
85 nmol/L (Normal range 20e109)

213

2016 Published by Elsevier Ltd.

CASE-BASED LEARNING

Abdominal discomfort was often experienced prior to the onset


of menstruation. Vaginal examination revealed a fixed retroverted uterus and tenderness in the pouch of Douglas. Her midluteal progesterone assay was 32 nmol/L. A transvaginal scan
revealed a right ovarian 6 cm endometriotic cyst. Mr PD was in
good health, worked as an accountant and had never fathered a
child. Semen analysis was normal.
 What is the optimal plan of management for this patient?
In view of this patients wish to pursue fertility investigations
and the findings on clinical examination it is advisable to offer
this patient a diagnostic laparoscopy and dye test to investigate
tubal patency. The most likely diagnosis here is endometriosis.
Endometriosis is a common finding in women of reproductive
age who present with pelvic pain and infertility. The patient
should also be counselled pre-operatively regarding ovarian
cystectomy as an ovarian cyst has been identified on transvaginal
ultrasound. Procedures such as adhesiolysis and diathermy to
endometriotic deposits should also be discussed with the patient
and consent obtained pre-operatively. All anticipated procedures
must be discussed pre-operatively and consent obtained.
 What impact does endometriosis have on reproductive
performance?
 Is surgery beneficial to patients with endometriosis?
Severe endometriosis can affect fertility by distorting pelvic
anatomy. However minimal/mild endometriosis appears to have
a negative effect on fertility. It is suggested that the peritoneal
environment is altered in the presence of endometriosis. While
both medical and surgical treatments are effective in treatment of
endometriosis-associated pain, the surgical approach is associated with lower recurrence rates as disease is removed rather
than suppressed as in medical therapy. For patients with infertility, however, medical treatment is ineffective in enhancing
fertility, while its role as an adjuvant therapy for assisted
reproductive treatment remains debatable.
At laparoscopy this patient was found to have a right ovarian
endometriotic cyst. A right ovarian cystectomy was performed.
Endometriotic deposits were noted in the pouch of Douglas and
on the left uterosacral ligament and these were ablated using
bipolar diathermy.
Evidence from two randomized trials has suggested a benefit
from surgical ablation. It is recommended that women with
minimal or mild endometriosis who undergo laparoscopy should
be offered surgical ablation or resection of endometriosis plus
laparoscopic adhesiolysis because this improves the chance of
pregnancy. Earlier studies indicated that women with moderate
or severe disease would have improved fecundity with the
removal of endometriotic implants. Surgery for severe and deep
endometriosis is unpredictably difficult and may be associated
with severe complications. Therefore a pre-operative ultrasound,
contrast enema and intravenous urogram may be required in
many cases, together with a full preoperative bowel preparation.
Surgery should be carefully planned and may require collaboration with a urologist or colorectal surgeon. ESHRE published
guidelines regarding management of women with endometriosis
in 2014 and advised that clinicians refer women with suspected
or diagnosed deep endometriosis to a centre of expertise that
offers all available treatments in a multidisciplinary context.
 What is the optimal management of ovarian
endometriomas?

Although not mandatory for the diagnosis of PCOS, transvaginal


ultrasound scanning (TVS) is usually advisable to exclude other
pelvic pathologies.
Mrs AT underwent TVS, which showed a small uterus with
endometrium thickened to 12 mm and bilaterally enlarged
ovaries (Total ovarian volumes of, 14 mL and 12 mL), each
containing more than 12 small follicles (2e9 mm) around the
periphery. There were no clinical or biochemical features of
LoCAH, Cushings syndrome or androgen secreting tumours. The
diagnosis at referral was therefore confirmed.
 In what ways does obesity affect reproductive
performance?
Very obese women with subfertility present a challenging
problem. Many are unable to comply with dietary or exercise
regimes prescribed for weight loss. These women will respond
poorly to common subfertility treatments and are more likely to
miscarry a pregnancy repeatedly, have a baby with congenital
defects or fetal macrosomia, or suffer an increased perinatal
morbidity from shoulder dystocia. They are also more likely to
require caesarean section and are many times more prone to all
its complications. Furthermore, gross obesity is associated with
PCOS, type 2 diabetes mellitus, gall-bladder disease, hypercholesterolaemia, hypertensive and coronary heart disease,
osteoarthritis and venous thromboembolic disorders. Therefore,
very obese, subfertile women who are sufficiently motivated, but
unable to achieve target body mass indices (BMI) should be
referred to specialists who can assess their eligibility for antiobesity drugs (Orlistat a Lipase inhibitor; or Sibutramine,
which is a serotonin and noradrenaline reuptake inhibitor) or
obesity (bariatric) surgery.
Mrs AT was advised to update her rubella immune status, and
encouraged to keep taking Folic acid. She was also advised to
reduce her weight towards a BMI target of 30. Metformin a
biguanide, was prescribed and the patient was referred to a
dietician. She declined fertility counselling. Metformin decreases
gluconeogenesis and increases peripheral utilisation of glucose
by increasing insulin sensitivity. Metformin is the usual first
choice treatment for obese PCOS patients because of its additional value in restoring ovulation in the majority of cases. Her
progress was however slow, but for her motivation; she became
eligible for adjunctive treatment with Orlistat. Ten months later,
she returned to the gynaecology clinic having achieved a BMI of
32, and a regular menstrual cycle. Mid-luteal progesterone assay
showed evidence of ovulation. She became pregnant three
months later following which she was advised to stop her medications and book early for antenatal care.

Case 4
A couple were referred to a tertiary centre with a history of
inability to conceive of 24 months duration. Both were 39 years
old and in relatively good health. Neither smoked cigarettes and
alcohol intake for both was minimal. Mrs CD had a regular
menstrual cycle, an up-to-date normal cervical smear test, body
mass index (BMI) of 27, was rubella immune and had screened
negative for Chlamydia infection. She complained of severe
dysmenorrhoea predominantly on days one to three of her cycle
with acute exacerbation on day one. The pain is often associated
with gastrointestinal symptoms of vomiting and diarrhoea.

OBSTETRICS, GYNAECOLOGY AND REPRODUCTIVE MEDICINE 26:7

214

2016 Published by Elsevier Ltd.

CASE-BASED LEARNING

 What advice would you give this 39 year old woman with a
diagnosis of endometriosis and right tubal blockage?
Two main surgical strategies have been used; including
excision biopsy or ablation. Medical therapy or simple aspiration and irrigation of endometrioma are ineffective, with
recurrence rates greater than 80% at 6-month follow-up.
Removal of the cyst lining, by either stripping or sharp excision of the capsule is frequently easy and is the optimum
treatment, although, when impossible, cyst-wall ablation gives
comparable results. Following adhesiolysis, irrigation and suction procedure, the ovarian capsule is identified around the cyst
opening. Once the plane of cleavage is found the cyst wall is
easily stripped from the ovary. Occasionally, it may be necessary to incise at the centre of the cyst in order to identify a
cleavage plane. Care must be taken to preserve ovarian tissue
and closure of the ovary is not proven to be necessary. Too
superficial diathermy ablation may result in inadequate treatment, whereas over-enthusiastic treatment may destroy ovarian
cortex and reduce its function.
The patient should be advised to proceed with in vitro fertilization treatment. ART is effective in the treatment of infertility
in patients with endometriosis. There are reports of poorer
ovarian responses to stimulation and poorer outcomes of ART
cycles in women with advanced endometriosis. A luteal phase,
GnRH analogue long protocol has been shown to improve live
birth rate.
This patient proceeded to IVF treatment which unfortunately
was unsuccessful for her. She then decided to explore the option
of adoption.
A

Johnson NP, Mak W, Sowter MC. Surgical treatment for tubal disease
in women due to undergo in vitro fertilisation. Cochrane Database
Syst Rev 2004. Issue 3. Art. No.:CD002125.
Marcoux S, Maheux R, Berube S, The Canadian Collaborative
Group on Endometriosis. Laparoscopic surgery in infertile
women with minimal or mild endometriosis. N Engl J Med
1997; 337: 217e22.
Menon DK. Successful treatment of anabolic steroid-induced azoospermia with human chorionic gonadotropin and human menopausal gonadotropin. Fertil Steril 2003; 79(suppl 3): 1659e61.
NICE guideline. Investigations of fertility problems and management
strategies. In: Moody Jane, ed. Fertility assessment and treatment
for people with fertility problems. Guideline developed by the National Collaborating Centre for Womens and Childrens Health.
London: RCOG Press, 2004; 39e56.
Vanky E, DE Zegher F, Diaz M, Ibanez L, Carlsen SM. On metformins
potential to prevent preterm delivery in women with polycystic
ovary syndrome - an epi-analysis. Acta Obstet Gynecol Scand
2012 Sep 24; http://dx.doi.org/10.1111/aogs.12015 (Epub ahead of
print).

Practice points
C

FURTHER READING
Balen AH. Management e diagnosis and treatment. In: Infertility in
practice. 3rd edn. London: Informa Healthcare, 2008; 123e288.
Camus E, Poncelet C, Gofnet F, et al. Pregnancy rates after in-vitro
fertilization in cases of tubal infertility with and without hydrosalpinx: a meta-analysis of published comparative studies. Hum
Reprod 1999; 14: 1243e9.
Dover RW, Torode H. Endometriomas: a review of modern management. Gynaecol Endosc 2000; 9: 219e26.
Dunselman GA, Vermeulen N, Becker C, et al. ESHRE guideline:
management of women with endometriosis. Human Reprod 2014;
29: 400e12. http://dx.doi.org/10.1093/humrep/det457. Epub 2014
Jan 15.

OBSTETRICS, GYNAECOLOGY AND REPRODUCTIVE MEDICINE 26:7

215

Good history and examination are essential: subfertility


frequently co-exists with other morbidities, some of which might
be life-threatening.
In the absence of co-morbidity, 95% of young couples will achieve
a natural pregnancy at the end of the second year of trying.
In all cases of severe oligozoospermia, or azoospermia, efforts
must be made to find causative factors.
PCOS accounts for over 70% of cases of anovulatory subfertility:
diagnosis involves excluding late-onset CAH, Cushings syndrome
and androgen secreting tumours.
Women with hydrosalpinx or hydrosalpinges detectable on ultrasound scan should be advised to have these excised or
occluded, if not amenable to repair, in order to enhance chances
of IVF.
Subfertile women with endometriosis should be advised to undergo excisional or ablative surgery to improve chances of
pregnancy.

2016 Published by Elsevier Ltd.

SELF-ASSESSMENT

Self-assessment questions
Questions

D)
E)
F)
G)
H)

Question 1 (SBA)
A 51-year-old woman with a BMI of 44 is diagnosed with an
early stage endometrial cancer at hysteroscopy performed for
chaotic perimenopausal bleeding. She attends the gynaecology
oncology clinic to discuss her condition and plan her treatment. She is very anxious and has searched extensively on the
internet for information about endometrial cancer before
meeting with you. Which one of the following statements is
true regarding her potential treatment?
A) Adjuvant radiotherapy provides no survival benefit in
women with early stage endometrial cancer.
B) Pelvic lymphadenectomy has an established therapeutic
benefit in women with early stage, low grade endometrial
cancer.
C) Systemic progestogens have a role in adjuvant treatment
following surgery for endometrial cancer.
D) For young women, in whom complete response has been
achieved with systemic progestogens, follow up with 6
monthly hysteroscopies should be recommended when
child bearing is complete.
E) Women with menopausal symptoms following surgery for
endometrial cancer should never be prescribed Hormone
Replacement Therapy.

I) Vitamin D
i) Occasionally used for symptomatic treatment of PCOS
symptoms in difficult or refractory cases, however long
term use can lead to bone demineralization, hot flushes,
and atrophic vaginitis.
ii) Indicated as an adjunct treatment to protect long-term
health in patients with premature ovarian failure.
iii) Therapy used to treat children with precocious puberty
due to radiation.
iv) Increases the ovulation rate by decreasing circulating
androgen levels.
Question 4 (EMQ)
Which one of the following ovarian pathologies best fits the
descriptions given below?
A) Benign immature teratomas
B) Hypogonadotropic hypogonadism
C) Polycystic ovarian syndrome
D) KISS1R mutation
E) Germ cell tumour
F) Dysfunctional uterine bleeding
G) Ovarian cyst
H) Kisspeptin 1 mutation
I) Gonadal dysgenesis
J) Malignant immature teratomas
K) MKRN3 mutation

Question 2 (SBA)
Which one of the following statements regarding risk factors
for developing endometrial cancer is correct?
A) Obesity increases the risk of type 2 endometrial cancers.
B) The BRCA1 mutation is an established risk factor for
endometrial cancer.
C) Tamoxifen use is associated with an increase in the risk of
endometrial cancer and women prescribed this medication
should have regular screening for endometrial pathology.
D) Hysterectomy and bilateral salpingo-oophorectomy is not
of benefit to reduce cancer risk for women with confirmed
HNPCC.
E) Endometrial cancer is the most common female cancer in
the United Kingdom.

i) Associated with an increased risk of gestational diabetes


and pregnancy complications.
ii) The aetiology of around 20% of cases of primary
amenorrhoea.
iii) The most common cause of peripheral precocious puberty
in girls.
iv) Loss of function mutations in this gene have been associated with central precocious puberty.

Question 3 (EMQ)
The following agents may have roles in managing peripubertal gynaecological problems. For each of the scenarios
below, choose drug that best fits the description given:
A) Medroxyprogesterone
B) Norethindrone
C) Cyproterone

Question 5 (SBA)
1. A 28-year-old patient with PCOS-associated infertility is
undergoing her second round of IVF treatment. During the first
cycle she experienced severe OHSS, followed by a miscarriage
of a twin pregnancy. She is anxious to avoid the possibility of
OHSS occurring again. Which one of the following strategies
would NOT be helpful to reduce the risk of OHSS?
A) Using low-dose stimulation protocol IVF
B) Follicular monitoring
C) Utilising GnRH analogue cycles rather than GnRH
antagonists
D) Utilising progesterone instead of HCG for luteal support

Catherine Aiken PhD MRCOG is a Clinical Lecturer in Obstetrics


and Gynaecology at Addenbrookes Hospital, Cambridge, UK.
Conicts of interest: none declared.

OBSTETRICS, GYNAECOLOGY AND REPRODUCTIVE MEDICINE 26:7

Flutamide
GnRH analogues
Metformin
Spironolactone
Growth hormone

224

2016 Published by Elsevier Ltd.

SELF-ASSESSMENT

2) A 28-year-old woman wishes to conceive with her HIV ve


male partner. She has had two early miscarriages with her
previous partner, which she conceived without difficulty.
Her partner has normal semen analysis.
3) A 30-year-old woman is planning her third round of IVF.
The first two cycles had to be cancelled prior to egg
collection due to the development of OHSS.

E) Cryopreserving embryos and deferring transfer to another


cycle
Question 6 (SBA)
The oocyte is orders of magnitude larger in size than a mature
spermatozoan, as it provides the almost all of the protoplasm
of the new zygote. However, some important components
derive from the spermatozoan. Which one of the following
components of the zygote is acquired from the spermatozoan?
A) Yolk proteins
B) Ribosomes
C) Centrosome
D) Cortical granules
E) Mitochondria

Question 10 (SBA)
A 6-year-old girl is referred to the gynaecology clinic because
of concerns from her parents and GP regarding the development of secondary sexual characteristics, including pubic hair
and breast development. Which one of the following investigations would NOT form part of your work-up for this
patient?
A) Pelvic ultrasound scan
B) Bone age assessment
C) MRI head
D) Serum gonadotrophin levels
E) Diagnostic laparoscopy

Question 7 (SBA)
A 22-year-old woman with a BMI of 36 is referred to the
reproductive medicine clinic by her General Practitioner for
advice about her irregular periods. At her initial clinic visit,
several investigations are performed. Which of the following
findings would not be in keeping with the possible diagnosis
of polycystic ovarian syndrome?
A) A free androgen index (FAI) of 5.6
B) Clinically evident hirsutism across the chest and upper back
C) First menses at 8 years
D) No cortisol suppression during overnight dexamethasone
suppression test
E) Endometrial thickening of 16 mm on transvaginal ultrasound scan

Self-assessment answers
Answer 1
A. Adjuvant radiotherapy provides no survival benefit in
women with early stage endometrial cancer.
No study of adjuvant radiotherapy has demonstrated significant survival benefit for women with at low or intermediate
risk of recurrence. Whilst radiotherapy has been shown to
reduce the risk of recurrence (relative risk reduction 72%) this
does not translate into a reduction in the risk of death for these
women. The ASTEC trial, a multinational study of 1400 patients with early stage endometrial cancer, failed to show any
benefit from lymphadenectomy in terms of overall survival or
disease-free survival. A study of more than 500 patients
(Benedetti Panici P, Basile S, Maneschi F, et al.) reached
similar conclusions. Critics have highlighted several potential
flaws with the ASTEC study, as outlined in the main text, and
debate continues, however present evidence does not support
a therapeutic advantage of lymphadenectomy for women with
early stage disease. Systemic progestogens have a role in the
management of women unfit for other types of treatment or in
the palliation of advanced disease however there is no role for
progestogens in adjuvant therapy due to the often partial and
short lived effect of hormonal treatment. There is no evidence
that oestrogen only HRT adversely affects survival in women
following completion of treatment management for early stage
disease and as such, oestrogen replacement may be prescribed
for those women experiencing distressing menopausal symptoms following appropriate discussion regarding risks and
benefits.

Question 8 (SBA)
A 38-year-old woman and her 44-year-old partner attend the
reproductive medicine clinic with secondary infertility of 5
years. During the investigations, the male partner admits to
using performance-enhancing anabolic steroids to aid his
career as a professional rugby player. Which of the following
health risks should you warn him about?
A) Testicular cancer
B) Liver tumours
C) Reduced libido
D) Increased bone demineralisation
E) Migraines
Question 9 (EMQ)
Chose the fertility treatment option that is most suitable for the
cases outlined below:
A) In vitro fertilisation
B) In vitro maturation
C) Gamete intra-fallopian transfer
D) Donor insemination
E) Intrauterine insemination
F) Intra-cytoplasmic sperm injection
G) Ovulation induction
H) Ovarian drilling

Answer 2
A. Obesity increases the risk of both type 1 and type 2
endometrial cancers.
Obesity is a significant risk factor for endometrial cancer and is
strongly implicated in the rising incidence of the disease. Whilst
obesity is predominantly associated with type 1, oestrogen

1) A 34-year-old woman and her partner have experienced


primary infertility for the past 3 years. The woman has
regular cycles and patent tubes. However semen analysis
shows her partner to have oligoasthenoteratozoospermia.

OBSTETRICS, GYNAECOLOGY AND REPRODUCTIVE MEDICINE 26:7

225

2016 Published by Elsevier Ltd.

SELF-ASSESSMENT

Answer 5
C. Utilising GnRH analogue cycles rather than GnRH
antagonists
Use of GnRH antagonists for pituitary suppression have
been associated with lower risk of OHSS by 40% and cycle
cancellation compared with GnRH agonist protocols. The
other options are all associated with a lower risk of developing
OHSS.

dependent, endometrial cancers, it also increases the risk of


type 2 tumours. BRCA1 carriers who do not use tamoxifen do
not have an increased risk of endometrial cancer compared to
the general population. While tamoxifen use does increase the
risk of endometrial cancer (2e5 times), there is no evidence that
screening of asymptomatic women is beneficial. Symptomatic
women should be promptly investigated by hysteroscopy and
endometrial biopsy. Women with confirmed HNPCC have a 40
e60% increased risk of endometrial cancer and no effective
screening strategy has been established. It is therefore reasonable to offer prophylactic surgery to these women on completion of their family. Endometrial cancer is the most common
gynaecological cancer in the UK however breast cancer remains
the female cancer with the highest incidence.

Answer 6
C. Centrosome
While the oocyte is the major contributor to the new
conceptus, the spermatozoan provides several key components including the male pronculeus and the centrosome.
Answer 7
D. No cortisol suppression during overnight dexamethasone suppression test
The other options here could all be in keeping with the
diagnosis of PCOS, however failure to suppress cortisol with
an overnight dexamethasone suppression test implies a
possible diagnosis of Cushings syndrome and therefore warrants a referral to endocrinology before proceeding with
treatment.

Answer 3
i. E
These symptoms arise from oestrogen deficiency, which is
a risk of long-term GnRH use.
ii. I
Vitamin D supplementation is often prescribed along with
calcium to prevent osteoporosis in women with premature
ovarian failure.
iii. H
Children with precocious puberty due to radiation often
have specific deficiency of growth hormone.
iv. G
Metformin is a biguanide used in the treatment of type II
diabetes. It was first observed that women taking metformin
plus clomiphene had more frequent ovulation than women
taking clomiphene alone. Metformin is used for the induction
of ovulation.

Answer 8
B. Liver tumours
Prolonged use of anabolic steroids can cause abnormal
liver enzymes, cholestasis or liver tumours. Increased libido is
likely with anabolic steroid use, but is concurrent with suppressed spermatogenesis.
Answer 9
1. F. Intra-cytoplasmic sperm injection
Sperm can be retrieved in this situation and injected into
the cytoplasm of the egg. The success rates of this procedure
are similar to those of IVF.
2. E. Intrauterine insemination
Use of this technique would allow sperm washing to reduce
the risk of HIV transmission to the female partner. From the
information given, there does not appear to be any additional
need for any fertility treatment for either partner.
3. B. In vitro maturation
This technique allows the collection of immature eggs from
the ovaries, which are then matured in vitro prior to fertilization. It reduces the risk of developing OHSS, but it is also
likely that there will be fewer eggs collected.

Answer 4
i. C
Polycystic ovarian syndrome has implications for reproductive and metabolic health throughout life, and this is
important to bear in mind during pregnancy.
ii. B
The most common causes of primary amenorrhoea are
gonadal dysgenesis (50%), hypogonadotropic hypogonadism
llerian agenesis (15%), transverse vaginal septum
(20%), mu
or imperforate hymen (5%), and pituitary disease (5%).
iii. G
The most common cause of peripheral precocious puberty
in girls is a large functional ovarian follicular cyst.
iv. K
Familial cases of central precocious puberty have been
attributed to paternally transmitted inactivating mutations of
the MKRN3 gene. Gain of function (not loss of function) mutations in Kisspeptin and KISSR are also associated with precocious puberty.

OBSTETRICS, GYNAECOLOGY AND REPRODUCTIVE MEDICINE 26:7

Answer 10
E. Diagnostic laparoscopy
The other investigations listed will often form part of the
recommended panel to diagnose precocious puberty. An examination under anaesthesia may also be indicated, but laparoscopy would rarely need to be performed.

226

2016 Published by Elsevier Ltd.

Vous aimerez peut-être aussi